NAVLE Question of the Day

¡Supera tus tareas y exámenes ahora con Quizwiz!

Fenbendazole Treat this whipworm infection with fenbendazole. Can use milbemycin oxime as a preventive for chronic recurring cases, but check dog's HW test as this drug is also a filaricidal. Also think of whipworms in ruminants and pigs.

The results of a routine fecal examination from a seven-year-old male neutered mixed breed dog is shown below. What is the treatment of choice? A - Fenbendazole B - Diethyl carbamizine C - Metronidazole D - Piperazine E - Ivermectin

Vitamin A deficiency This is an aural abscess, which can occur secondarily to vitamin A deficiency. Captive terrestrial box turtles are most at risk, usually due to diets containing little vitamin A. Other presentations of hypovitaminosis A include froth from nose (a sign of respiratory disease) and renal disease.

Which one of the following choices can be an underlying cause for the problem seen in this turtle? A - Septicemic cutaneous ulcerative disease (SCUD) B - Vitamin A deficiency C - Nutritional secondary hyperparathyroidism D - Mycoplasmosis E - Pasteurella multocida

Closed reduction and ehmer sling Conservatively treat coxofemoral luxation with closed reduction and an Ehmer sling. Typically presents as a non-weight-bearing lameness, usually after blunt force trauma. Craniodorsal luxation of the femoral head is most common. Tx: closed or open reduction. Closed reduction and Ehmer sling if injury <48 h old, patient is stable for anesthesia, and hip structure is normal. 50% success rate. Open reduction if closed reduction is unstable or has failed. Carefully monitor post-reduction for irritation wounds and ischemia caused by ehmer sling.

A 12-year-old male neutered West Highland White terrier is presented after he disappeared from home for six hours and returned with a left hind limb lameness. A pelvic radiograph taken on presentation is shown below. What is the most appropriate initial treatment? A - Non-steroidal anti-inflammatory medication and rest B - Femoral head and neck excision C - Tibial tubercle transposition and Robert-Jones bandage D - Closed reduction and Ehmer sling E - Triple pelvic osteotomy

Infectious bursal disease (IBD)

A birnavirus causes which one of these avian diseases? A - Infectious laryngotracheitis B - Infectious bursal disease C - Marek's disease D - Newcastle disease E - Infectious bronchitis

You can perform the procedure with proper analgesia The AVMA position on velveting is that is must be conducted humanely and within the bounds of a VCPR. Velveting is the amputation of live, growing, immature antlers from cervids in the 55th-65th days of growth. Canadian resources support a similar position. Velvet antlers are innervated and vascularized tissues. Velvet peels off later in the season as the antler matures. Moos and elk have velvet antlers which grow form small immature stub

A client who raises North American farmed deer and elk asks you to remove off the entire soft antler of several animals in the first 55-65 days of growth, a practice call velveting. What should you tell the client? A - velveting is unnecessary; antlers fall off on their own at the end of mating season B - practice is illegal and sale of velvet antlers is internationally banned C - you can perform the procedure with proper analgesia D - velveting is unwarranted; velvet falls off on its own after two to three months E - velvet antler removal is painless, but adequate restraint is very important to minimize stress in captive-raised cervids

Amblyomma spp tick Vectors for ehrlichia ruminatium, causative agent of heartwater disease in ruminants. Clinical progression of this case and the pericardial effusion seen on necropsy are classical for acute heartwater disease. Animals may just be found dead with peracute disease. Definitive diagnosis typically requires necropsy with microscopic evaluation of stained brain tissue. Disease in endemic in parts of Africa and the Caribbean, but is considered a FAD in Americas. Dermacentor vector for Babesia caballi, Anaplasma marginale, RMSF, powassan virus, Q fever, tularemia Face flies vector for Thalazia and Mycoplasma bovis Horse flies can be vectors for bovine leukosis virus, th cause of enzootic bovine leukosis and lymphosarcoma.

A dead goat is presented to a veterinarian in the Caribbean. The farmer reports that the goat hadn?t been eating well yesterday, then this morning was breathing hard and walking with a strange high-stepping gait.She eventually went down and started paddling with her head twisted back just before death. Necropsy reveals the following finding (blue arrow) adjacent to the heart:He is worried about his other goats. What vector is responsible for transmitting the infectious agent most likely responsible for this goat's condition? A - Musca autumnalis (the face fly) B - Damalinia caprae lice C - Amblyomma spp. ticks D - Tabanids (e.g., the horse fly) E - Dermacentor spp. ticks

Mannheimia haemolytica Mannheimia haemolytica pneumonia typically has a cranioventral distribution - as demonstrated by the hepatized lung seen in the bottom half of this image. M. haemolytics and Pasteruella multocida are commensals in the upper airway of sheep. M. haemolytica of sheep and goats is a different strain from that in cattle (type A2 vs A1). Infection is more likely if there is a primary viral or bacterial infection. Pneumonia with these pathogens is most common after recent stress or commingling with new animals. Prevention involves decreasing stress and exposure. There are currently no vaccines available for respiratory disease in sheep and goats and cattle vaccines have not been shown to be effective. Ovine progressive pneumonia caused chronic wasting with respiratory distress and occasional indurative mastitis. Necropsy findings include lungs with mottled gray and brown areas of consolidation and very enlarged mediastinal and tracheobronchial lymph nodes. Corynebacterium pseudotuberculosis caused caseous lymphadenitis is sheep and goats. Multifocal abscesses would be seen on necropsy. Dictyocaulus filaria can cause bronchitis or pneumonia in sheep and infecti

A farmer recently purchased a sheep from a salebarn. The sheep soon became ill, with progressive depression, lethargy, and inappetence. He had respiratory problems, breathing hard with abdominal effort, and died after three days with a frothy nasal discharge just prior to death. At necropsy, the lungs had the appearance shown below. What was the most likely cause of death? A - Dictyocaulus filaria B - Mannheimia haemolytica C - Perilla frutescens D - Ovine progressive pneumonia E - Corynebacterium pseudotuberculosis

Mediastinal lymphoma On the lateral thoracic radiograph, there is a mass in the cranial mediastinum with mild dorsal displacement of the trachea. The most likely diagnosis for a mediastinal mass in a dog with hypercalcemia in lymphoma. Radiographic interpretation: the cardiovascular structures are normal in size and shape. No abnormalities are seen within the pulmonary parenchyma. Radiographic interpretation: the cardiovascular structures are normal in size and shape. No abnormalities are seen within the pulmonary parenchyma. On the lateral projection there is a mass in the cranial mediastinum with mild dorsal displacement of the trachea. The mediastinum is widened on the V/D projection, and is wider than the vertebral column. The pleural space and musculoskeletal structures are normal.

A five-year-old female spayed Australian shepherd is presented for poor appetite and lethargy. Bloodwork reveals hypercalcemia. Urinalysis is normal. Abdominal ultrasonography is unremarkable. Thoracic radiographs are shown below. What is the most likely diagnosis? A - Idiopathic hypercalcemia B - Metastatic lung disease C - Hepatic parathyroid hormone-related protein releasing tumor D - Hypertrophic osteopathy E - Mediastinal lymphoma

Low dietary copper Low dietary copper levels causes bones of growing animals to be brittle because they do not develop the normal trabecular structure. Higher fracture rates are seem in Cu deficient than in Cu replete animals. Pathologic fractures are more common during periods of exertion, such as the morning round up. Rough handling exacerbates the number of fractures. Treatment of Cu deficiency includes copper glycinate injections, and feeding of mineral mixes with Cu. Copper boluses can also be given. In areas with high levels of molybdenum in the soil, Cu levels are often low. Mb interferes with absorption of Cu, hence Mb excess results in Cu deficiency. Pathologic fractures can be seen with hypovitaminosis D growing cattle, but this is unlikely in beef cattle on pasture, eps in the souther United States where there's plenty of sunlight. Occasionally seen in northern cows in winter or show calves housed primarily indoors. Calcium-deficient calves may also develop a similar syndrome of pathologic fractures.

A four-month-old calf from a small beef herd in Louisiana presents with a fracture of the right metatarsus. He was normal last night. The owner found the calf this morning after rounding up the herd for deworming. This is the fifth calf this season with fractured bones. One had a vertebral fracture, two calves had humeral fractures, and one other had a cannon bone fracture. Which one of the following choices is the most likely problem in this herd? A - Hypovitaminosis D B - Low dietary copper C - Low dietary protein D - High dietary phosphorus E - High dietary calcium

Antibodies against P2-myelin protein The top differential for this horse with cauda equina signs PLUS cranial nerve signs is polyneuritis equi. Horses with this condition can have circulating antibodies against P2-myelin protein, but many false positives. Other diseases to rule out in this case would be equine herpesvirus-1 and equine protozoal encephalomyelitis. Unfortunately there is no definitive treatment and prognosis is poor for functional recovery.

A four-year-old stallion is presented with urine scald, fecal retention, tailhead rubbing, and a right head tilt. Rectal examination reveals atonia of the anus and rectum, however, no fracture is palpated. Which one of the following tests help provide evidence of the top differential diagnosis? A - Search the pasture for yellow star thistle B - Antibodies against P2-myelin protein C - Tibial muscle and nerve biopsies D - Cervicothoracic spinal radiography E - Immunoblot against DNA fragments from P. tenuis

Metabolic alkalosis Think if hypochloremic metabolic alkalosis due to hydrochloric acid (HCl) sequestion in the abomasum of a cow with a displaced abomasum. Abomasal hypomotlity ongoing HCL secretion into the abomasum and partial abomasal outflow obstruction all contribute. Metabolic alklosis can also be due to HCl loss in monogastric animals who vomit. In a similar way, metabolic acidosis can occur with HCO3 loss in saliva if animal cannot swallow or from losses in diarrhea.

A high-producing dairy cow that freshened three weeks ago is off feed. On physical exam her heart and respiratory rates are within normal limits. T=101.2°F (38.4°C) [N=100.4-102.8°F (38.0-39.3°C)] There is decreased rumen motility and a urine test for ketone bodies is positive. There is no evidence of mastitis and the uterus is clear of infection. On the left side, a high-pitched musical "ping" is audible via stethoscope during percussion over the ribs on a line between the elbow and tuber coxae (hip). What acid-base abnormality is most likely in this cow? A - Metabolic alkalosis B - Respiratory acidosis C - Depends on severity of displacement D - Metabolic acidosis E - Respiratory alkalosis

Did you use chlorine or other chemicals near the tank/equipment or do a recent water change with tap water? Chlorine is commonly used to disinfect tanks and equipment but is highly toxic to fish. This is a common problem because both chlorine and chloramine (type of chlorine stabilized by animation) are frequently found in municipal water sources. Chronic and subacute chlorine exposure result in signs as describe here: cloudy eyes, inflammation, and necrosis of the gills, excess mucus covering gills, and lethargy. Acute chlorine toxicosis can cause sudden death. There is a colorimetric test readily available from pool supply stores that measures chlorine; levels should be non-detectable when tested on site. If the sample of water needs to be transported for testing, use a sealed glass bottle with no air space; false negative are possible. Need to measure both free and total to get the most accurate results. Tx with dechlorination products (often containing sodium thiosulfate) available at pet stores and then recheck free and total chlorine levels. Increase aeration because of gill compromise. Addition of 1-3 ppt salt may also help with osmoregulatory issues.

A hobbyist fish owner is concerned because, within just a few hours, many of the fish in his freshwater tank have developed cloudy eyes, increased mucus, and are gasping at the surface. Fish also seem to be moving slowly. Based on this history, which of the following would be the most relevant question to ask? A - Did you use chlorine or other chemicals near the tank/equipment or do a recent water change with tap water? B - Were these fish treated with ivermectin, levamisole, or praziquantel? C - What are the levels of vitamin A, thiamine, and vitamin E in the feed, and how do you store the feed? D - How often do you measure ammonia levels in the tank water and what was the last reading? E - Do you feed frozen or live brine shrimp or bloodworms, and if so, a) how often are these fed and b) are they rinsed prior to use?

Mesenteric volvulus This in an example of mesenteric volvulus. Note the marked generalized gas distention of the bowel. Obstruction of blood supply to the intestine causes ileus, and the discoloration of the serosa seen in the second image. Mesenteric volvulus in uncommon and is nearly always fatal due to irreversible severe diffuse bowel wall ischemia. German shepherds, English pointers, and other large breed dogs are predisposed. The presentation can resemble gastric dilatation and volvulus - acute abdominal pain and distention, unproductive retching and shock A linear foreign body causes intestinal plication visible of radiographs and at exploratory laparotomy.

A nine-year-old female spayed German Shepherd is presented for non-productive retching and collapse. Physical examination reveals severe abdominal pain, tachycardia, pale mucous membranes, poor pulse quality, and tachypnea. The dog is stabilized with intravenous fluid therapy, and an abdominal radiograph is shown below. The second image reveals the findings on exploratory laparotomy. What is the most likely diagnosis? A - Linear foreign body B - Gastrointestinal ulceration C - Intestinal lymphosarcoma D - Intussusception E - Mesenteric volvulus

Disseminated intravascular coagulation (DIC) A pattern of low platelets, increased bleeding time, and across the board increased in aPTT, PT, and FDP tests suggests disseminated intravascular coagulation (DIC). DIC is not a disease in its own right. It is a complex hemostatic defect characterized by inappropriate activation of coagulation and deposition of fibrin within the microvasculature. The depletion of clotting factors and platelets due to this clot formation can result in widespread hemorrhage. If DIC is suspected it is essential to identify the underlying cause. Many disease can result in DIC, but the more common conditions include: sepsis, advanced heartworm disease, immune-mediated hemolytic anemia, anaphylaxis, envenomation, heat stroke, and severe fulminant pancreatitis. DIC is seen more commonly in dogs than in cats. Chronic DIC may be seen in dogs with hemangiosarcoma.

A nine-year-old male neutered German shepherd is presented with unchecked bleeding from a cut on the gums above the right canine tooth. The owner relates that the dog has lost weight and had an episode of collapse three days ago, but he recovered. On physical exam, the gums are pale with petechiae and ecchymotic hemorrhages. There is tachycardia and a palpable cranial abdominal mass. A coagulation profile shows the following: Thrombocytes= 82,533 per microliter. [N=200,000-900,000] Buccal mucosal bleeding time (BMBT), increased Activated partial thromboplastin time (aPTT), increased Prothrombin time (PT), increased Fibrin degradation products (FDPs), increased What disorder of coagulation best fits this pattern? A - Anticoagulant rodenticide toxicity B - Disseminated intravascular coagulation C - Von Willebrand's disease D - Hepatic insufficiency E - Idiopathic thrombocytopenia

Thyroid function, BUN/creatinine Evaluate thyroid function and measure BUN/creatinine to rule out underlying disease because this presentation of vision loss in an older cat suggests retinal detachment secondary to hypertension. Always perform a blood pressure measurement in a cat with retinal detachment. Chronic renal disease can lead to hypertension and retinal detachment. Remember that BUN/creatinine results should always be interpreted in light of a urine specific gravity. Hyperthyroidism can also cause hypertension. The physical exam, history, and a total T4 measurement will help evaluate for this condition. Other diseases associated with non-hypertensive retinal detachment include FIP, FeLV, FIV, cryptococcosis, and toxoplasmosis.

A nine-year-old male neutered domestic shorthair cat is presented with a four-week history of disorientation and bumping into things. The cat does not visually track a ball rolled in front of him, and has decreased pupillary light reflexes OU. On fundoscopic exam, the retinal vessels and disc are obscured (see image below). What testing is indicated to assess the two most common underlying causes of this condition in cats? A - Urinalysis, pancreatic function, fasting blood glucose B - Thyroid function, BUN/Creatinine C - Adrenal function, arterial pressure D - Hepatic function, serum globulins E - CSF analysis, CBC

Classical swine fever (CSF) Hemorrhages on the kidneys and other organs are characteristic of both classic swine fever (aka hog cholera) and African swine fever (ASF) African swine fever cannot be differentiated from classical swine fever based on clinical and postmortem signs alone. Confirmation is based on either PCR or ELISA antigen testing. Both are reportable diseases. Classical swine fever was last reported in North America in the 1970s. Erysipelas is characterized by fever, painful joins, and sometimes, urticarial diamond shaped skin lesions. Glaesserella parasuis occurs mainly in young piglets. Pigs with post-weaning multi-systemic wasting syndrome (PMWS) typically have enlarged, pale lymph nodes, growth retardation, wasting, and dyspnea. Mycoplasma hyopneumoniae is a common cause of pneumonia in adult pigs; present with a barking cough and slow growth.

A pig farm has several sick and dying adult pigs. Affected animals are febrile and depressed. Some seem constipated and others have diarrhea. A few are ataxic. A necropsy on one of the dead pigs shows widespread petechial and ecchymotic hemorrhages in the kidneys ("turkey egg kidneys"), bladder, spleen, and larynx. Which one of the following choices is the most likely diagnosis? A - Mycoplasma hyopneumoniae B - Postweaning multisystemic wasting syndrome C - Glaesserella parasuis (formerly Haemophilus parasuis) D - Classical swine fever E - Anthrax

PVN is 99% The trick with this kind of question is to pick an imaginary number of animals that you test, like 100, and filll out your 2x2 tablet from there. If prev is 5%, then either must be a 500/1000 cats with FeLV and 950 that are disease free. A 90% sensitive test will correctly call 40/50 positive and incorrectly call 5/50 negative. If 50/1000 animals are infected, then 950/1000 are disease free. Your 95% specific test will correctly call 902/950 disease-free and incorrectly call 48-950 positive.

A practice is using an FeLV test with a sensitivity of 90% and a specificity of 95%. Assuming the prevalence of feline leukemia in the area is 5%, what is the predictive value negative (PVN) of the test? A - 45% B - 88% C - 99% D - 75% E - 55%

Septic arthritis The duck has septic arthritis. There is diffuse soft tissue swelling of the left pelvic limb. There also appears to be intracapsular soft tissues swelling of the left tibiotarsal joint causing the joint space to widen asymmetrically. An osseous fragment is noted at the medial aspect of the joint. The articular margins of the tibiotarsus and the tarsometatarsus appear lytic and irregular. Egg-binding signs are nonspecific but lameness is not a key finding typically. Sometimes the bird will adopt a penguin-like posture. Egg binding is more often seen during the spring and summer.

A rescued Pekin duck is presented for non-weight bearing lameness of the left pelvic limb and fever. Which one of the following choices is the most likely diagnosis? A - Normal radiographs B - Panosteitis C - Septic arthritis D - Egg binding E - Bumblefoot

Inguinal hernia A common problem in pigs; treat surgically via midline skin incision, cranial to scrotum. Important to ligate and excise the vas deferens and blood vessels. Both inguinal rings should be closed to prevent herniation post-surgery. Prevent seroma formation by removing the tunic, cremaster muscle, and extra subQ tissue; surgically close any dead space. Intersex syndrome is described in pigs and goat, but look for male and female genitalia, Perineal hernia is more a problem of middle-aged pure-bred dogs. In cryptorchid animals, a testicle is retained, not protruding.

A seven-week-old pot-bellied pig is presented with a soft swelling ventral to the anus, between the hind limbs. Which one of the following is the most likely clinical diagnosis? A - Intersex syndrome B - Cryptorchidism C - Inguinal hernia D - Testicular torsion E - Perineal hernia

Immune-mediated thrombocytopenia This is immune-mediated thrombocytopenia, the most common cause of spontaneous bleeding in dogs. It may be primary (idiopathic, most common) or secondary to infection, neoplasia, or certain drugs. Most common in middle-aged females; cockers, poodles, and old english sheepdogs. Dogs with disseminated intravascular coagulation (DIC) have thrombocytopenia with a prolonged PT and aPTT.

A six-year-old female spayed cocker spaniel is presented with a two-day history of lethargy. Exam reveals petechiae and ecchymoses on the buccal mucosa, ventral abdomen, and inner pinnae. There is hyphema OD. Click the labwork icon to review hematology, blood chemistry, and urinalysis test results. Prothrombin time (PT) and activated partial thromboplastin time (aPTT) are normal. What is the most likely diagnosis? A - Immune-mediated thrombocytopenia B - Anticoagulant rodenticide toxicity C - Disseminated intravascular coagulation D - Von Willebrands disease E - Hepatic insufficiency

Thoracic radiographs to evaluate for congestive heart failure. This is a classic presentation for dilated cardiomyopathy; perform thoracic radiographs to evaluate for congestive heart failure. Perform electrocardiogram to better assess the cardiac rhythm. Use echocardiography to confirm the diagnosis. Consider veterinary point of care ultrasound (POCUS) to assess for pleural effusion and alveolar infiltrates. TX to control arrhythmia and diminish clinical signs. Therapy may include diuretics, positive inotropes, angiotensin converting enzyme inhibitors, and antiarrhythmics. Px is guarded, especially for Dobermans - with treatment, may live 6mo-2years. Large and giant breed are at increased risk, including Irish wolfhounds and great danes. Boxers can develop arrhythmogenic right ventricular cardiomyopathy, distinct from DCM. Dobermans have two genetic mutations that can lead to development of DCM: PDK4 and DCM2. Each mutation independently can lead to DCM: dogs affected with both at 30x more likely to develop DCM than a normal dog.

A six-year-old male neutered Doberman pinscher is presented for a one-week history of coughing, depression, and exercise intolerance. Exam reveals tachycardia with an irregularly irregular heart rhythm and pulse deficits. Once the dog is stabilized which initial diagnostic step is most appropriate? A - Ultrasound for pericardial effusion B - Abdominal focused assessment with sonography for triage, trauma, and tracking rapid (AFAST) for hemoabdomen C - Bronchoscopy for airway assessment D - Thoracic radiographs to evaluate for congestive heart failure E - CBC to identify anemia

Cervicothoracic C6-T2 Weak, hypotonic (LMN) forelimbs and spastic paresis (UMN) hindlimbs are signs of a cervicothoracic lesion. May see worse signs in the fores than hinds. This is the opposite presentation as Schiff-Sherrington syndrome (severe acute spinal cord trauma of region T3-L3), where in lateral recumbency, the thoracic limbs are rigid and extended and the pelvic limbs appear flaccid in comparison. However, pelvic limb reflexes are normal to increased, as would be expected with an upper motor neuron lesion. Can localized T3-L3 lesions by checking cutaneous trunci reflex - lesion is usually 1-2 vertebrae cranial to where the reflex disappears. With C1-C5 would expect UMN signs in all four limbs, usually worse in hinds. With a T3-L3, would see UMN hind limb signs and normal forelimbs.

A stray dog is presented after being hit by a car. The dog has hypotonic forelimbs and spastic paresis in the hinds. All four limbs have proprioceptive deficits and sensation loss-signs are worse in the forelimbs. Where is the lesion? A - Cervicothoracic: C6-T2 B - Cannot say without cutaneous trunci reflex results C - Lumbosacral L4-S3 D - Thoracolumbar T3-L3 E - Cranial cervical: C1-C5

Good prognosis for survival with surgery This dog has a gastric dilation/volvulus and based on the lactate level, he has a good prognosis for survival with surgery. Plasma lactate concentration is the most important prognostic indicator in dogs with GDV. Dogs with a lactate <6mmol/L on presentation have a 99% survival rate with surgery. A lactate >7.4mmol/L at presentation predicts gastric necrosis and decreasing prognosis with high accuracy. However, comparing pre and post treatment lactate levels may be more predictive of survival than an initial one time lactate measurement. A 50% or greater reduction in lactate within 12 hours of presentation carrier an improved prognosis. GDV is an acute enlargement and rotation of the stomach, mostly seen in large and giant breed dogs. The enlarged stomach compresses the caudal vena cava, leading to hypovolemia; pushes on the diaphragm, causing respiratory dysfunction; and

A ten-year-old male neutered Irish setter is presented with an acute onset of severe abdominal distension, non-productive retching, and collapse. Physical exam reveals a tense, distended abdomen. A right lateral abdominal radiograph is shown below. Click the labwork icon to review hematology and serum biochemistry results. Which choice is the best message to communicate to the owner about next steps to take in light of these findings? T - 101.6, HR = 140bpm, RR = 43 bpm A - Dog will need a splenectomy but not gastropexy B - Gastric decompression via stomach tube will resolve abnormality C - Abdominocentesis is necessary to address hemoabdomen D - Good prognosis for survival with surgery E - The prognosis is poor, and humane euthanasia may be the best course

Multiple myeloma (MM) This is multiple myeloma, a malignancy of plasma cell origin. Malignant plasma cells arise from bone marrow and produce immunoglobulins, resulting in hyperglobulinemia. Hypergolbulinemia causes hyperviscosity and many of the clinical signs seen with MM. Clinical signs include lethargy, weight loss, bone pain or fractures, CNS signs, capillary bleeding, visual disturbances (due to retinal hemorrhage), organomegaly, and heart murmurs (due to hyperdiscosity) DX requires two or more of the following: Monoclonal gammopathy on serum electrophoresis Bence Jones proteinuria Lytic bone lesions on radiographs Plasma cell infiltration of the bone marrow

A ten-year-old male neutered West Highland white terrier is presented with a recent history of polyuria/polydipsia, lethargy, and weight loss. Physical exam reveals a grade 2/6 systolic heart murmur and mild muscle wasting along the lumbar spine. Significant lab abnormalities include pancytopenia, hyperglobulinemia, hypercalcemia, and isosthenuria. What is the diagnosis? A - Multiple myeloma B - Osteosarcoma C - Aplastic anemia D - Systemic lupus erythematosis E - Babesia canis

Joint neoplasia This is neoplasia involving the elbow joint. Radiographic interpretation: there is geographic lysis involving the distal humerus, proximal ulna, and to a lesser degree the proximal radium. The areas of bone loss are multifocal with relatively well circumscribed borders. The anconeal process is absent. There is a large soft tissue swelling centered around the left elbow. Ultrasound: there is a heterogenous hyperechoic lesion surrounding the caudal aspect of the elbow and also extending medially and laterally. There are severe defects within the bone and the mass lesion extends into the bone. Fine needle aspirate and cytology of the lesion revealed mesenchymal proliferation and marked atypia with increased plasma cells and recent hemorrhage - probable sarcoma.

A ten-year-old male neutered mixed breed dog is presented with a two month history of progressive left thoracic limb lameness. Radiographs and an ultrasound of the affected limb are shown below. What is the most likely diagnosis? A - Degenerative joint disease B - Septic arthritis C - Joint neoplasia D - Rheumatoid arthritis E - Fragmented coronoid process

Histoplasmosis This is histoplasmosis causing respiratory and gastrointestinal signs. Histoplasma capsulatum infection is seen in the river valleys of the central US, associated with bird and bat droppings in soil. The big DDX is blastomycosis, also seen in the same area, but with a respiratory presentation. Histoplasmosis is more a chronic diarrheal and respiratory disease in dogs and a respiratory disease in cats. Histoplasma organisms are much smaller than Blastomyces and are difficult to detect with routine H&E stain, unlike Blastomyces organisms. Use special stains to see yeast forms in macrophages and giant cells. If cytology is not useful consider antigen testing of urine, serum, or CSF. Cross-reactivity with blastomycosis may occur. Think of coccidioidomycosis in the arid and semiarid southwestern US, Mexico, and Central America. Spores are carried on dust and inhaled. Epidemics may occur after dust storms or excavation. Organisms cary in size and appear as spherules with a double-contoured wall. Mature spherules contain tiny endospores. Aspergillosis most commonly causes nasal signs in dogs, such as epistaxis, nasal congestion, and depigmentation of the nasal pla

A three-year-old male neutered Labrador retriever from the Ohio river valley basin is presented with a six-week history of mixed bowel diarrhea, cough, lethargy, and weight loss. A lymph node aspirate reveals round to ovoid (1-4 ?m) encapsulated structures inside macrophages and giant cells as shown in the image below. What is the diagnosis? A - Histoplasmosis B - Aspergillosis C - Blastomycosis D - Cryptococcosis E - Coccidioidomycosis

Conium maculatum (Poison hemlock) At least eight toxic piperidine alkaloids have been isolated from poison hemlock. Coniine is found in seeds and the mature plants; g-coniceine is found in young growing plants. Poison hemlock is toxic to all livestock and humans. Signs of toxicity develop one to two hours after ingestion and are usually fatal. Signs include nervousness, trembling, weakness (especially of the hind limbs), weak pulse, irregular heart rate, recumbency, coma, and death. A mousy odor exuding from the urine and breath is pathognomonic. Ingestion of poison hemlock during gestation causes arthrogryposis and other congenital defects in cattle, goats, and pigs.

A two-year-old Angus cow is presented with weakness of the hind limbs and a staggering gait. On physical exam, the heart rate and respiratory rate are slow and irregular, and the pupils are dilated. The cow's breath and urine smell like the odor of mouse urine. Suddenly the cow's pulse becomes rapid and thready, she collapses, and dies of apparent respiratory failure. Which one of the following plants is most likely to have caused this spectrum of clinical signs? A - Tetradymia spp (Horsebrush) B - Conium maculatum (Poison hemlock) C - Pinus ponderosa (Western yellow pine) D - Centaurea spp (Knapweed, Yellow star thistle) E - Veratrum spp (False hellebore, Skunk cabbage)

H1 histamine receptor antagonist; corticosteroids This is a well-differentiated mast cell tumor, which should be treated with an H1 histamine receptor antagonist and a corticosteroid prior to surgery. Treat pre-surgically to shrink the tumor, minimize degranulation, and block the effects of histamine and other vasoactive mediators. Risk of histamine-induced gastric ulceration can be decreased with proton pump inhibitors +/- H2 histamine blockers. Dx before surgical excision: CBC, serum chemistry panel, urinalysis, and regional lymph node aspiration. CBC changes can include eosinophilia, basophilia, and regenerative or non-regenerative anemia. If there is lymph node metastasis, tumor recurrence, or peritumoral edema/bruising also consider abdominal ultrasound +/- hepatic and splenic aspirates and bone marrow cytology. Complete excision requires at least two cm lateral margins and one fascial plane. Mast cell tumors can occur in any age or breed. Predisposed breeds include boston terrier, pug, shar-pei, staffordshire bull terrier, golden and labrador retrievers, and jack russell terriers.

An eight-year-old female spayed Boston terrier is presented with a solitary well-circumscribed mass two-cm in diameter on the left hip area, shown below. The owner reports that the mass occasionally appears inflamed, and that the dog has a history of allergies. A fine needle aspirate of the mass stained with Diff-Quik is also shown below. Based on the clinical presentation, cytological findings, and presumptive diagnosis, the owner opts for surgical resection. What pre-operative treatment is indicated? A - Adrenergic agonist; bronchodilators B - Bactericidal beta-lactam antibiotics; calcium channel blocker (vasodilator) C - H1 histamine receptor antagonist; corticosteroids D - No pre-operative treatment is indicated E - Nonsteroidal anti-inflammatory analgesics; H2 histamine blocker

Hypothyroidism Tail alopecia and tragic facial expression are classic signs of hypothyroidism in dogs. Other areas of bilateral alopecia that are common in hypothyroidism are the ventral and lateral trunk, caudal thighs, ventral neck, and dorsum of the nose. Hyperadrenocorticism commonly has truncal alopecia and also presents with obesity. It is more common in toy dogs breeds.

An obese eight-year-old Labrador Retriever is presented for annual vaccinations. The owners claim that they have restricted his diet and try to exercise him but he still keeps the weight on. There is alopecia on the dorsum of his tail and he has an unusual worried facial expression. Which one of the following choices is the most likely diagnosis? A - Sarcoptic mange B - Hypoadrenocorticism C - Hyperadrenocorticism D - Hypothyroidism E - Male pattern alopecia

Left side of the neck Palpate and visualize (as it passes down) the nasograstic tube in the left side of the neck. The esophagus runs down the left side of the horse's neck. It is NOT possible to see or safely palpate it in the larynx or oropharynx. It is critical to confirm proper placement prior to administration of fluids or medications through the NG tube. Other methods to confirm that the NG tube is in the stomach: Blow into the end and have an assistant auscult the stomach with a stethoscope Most patients cough if the tube is placed into the trachea; however, this is NOT fool-proof. Sedated or very sick patients, or those in which a very small diameter tube is used, may not cough. Negative pressure upon aspiration is a sign of being in the stomach. However, can also get negative pressure if the tube is up against soft tissue (in the lung or esophagus) so is not definitive.

At what location can palpation and visualization confirm proper nasogastric tube placement in the horse? A - Oropharynx B - Left side of the neck C - Right jugular groove D - It is not possible to confirm this way E - Larynx

Spirocerca lupi

During the necropsy of an eight-year-old mixed breed dog from the Southern United States, reactive granulomas in the esophagus containing bright red worms, 40 mm to 70 mm long are found. Which choice is the most likely diagnosis? A - Spirocerca lupi B - Ollulanus tricuspis C - Physaloptera spp. D - Haemonchus placei E - Gastrophilus spp.

Infectious laryngotracheitis Blood in the trachea and this history suggest infectious laryngotracheitis (ILT). A highly contagious herpesvirus infection, ILT is characterized by respiratory signs (rales, severe dyspnea, coughing). In severe forms, mortality can reach 50% typically due to occlusion of the trachea by blood, mucus, or caseous exudates. In most states ILT is reportable. Infectious coryza is a milder respiratory disease, with swelling around the eyes and head, sneezing, nasal discharge. Newcastle disease is characterized primarily by respiratory signs. Severe forms include depression, neurologic signs or diarrhea. Look for GI hemorrhage with most severe form, viscerotropic velogenic newcastle disease (VVND), which is also reportable. With fowlpox, only a few birds typically affected with scab-like lesions around head. See only depression before death with Marek's disease. Some birds may have characteristic unilateral paresis (one leg forward, one leg back). On necropsy, see lymphoid tumors in organs and enlarge nerves.

Half of the flock from a local broiler poultry farm has died in the last ten days. Examination shows chickens that are gasping and coughing, extending their necks to inhale. Some have blood-stained beaks. There are no obvious neurologic signs. Necropsy finds hemorrhage in the airways, as shown below. What is the most likely presumptive diagnosis? A - Infectious laryngotracheitis B - Fowl pox, diphtheritic form C - Viscerotropic velogenic Newcastle disease D - Infectious coryza E - Marek's disease

Treat affected cows with 2 classes of dewormer The correction action is to treat affected cattle with 2 classes of dewormer. An abomasum with a cobblestone or Moroccan leather appearance in the fall in the Southern US is pathognomonic for Type II ostertagiasis caused by O. ostertagi. This finding in a heard with routine deworming suggests anthelmintic resistance. Current recommendations are to dewormer with two different classes of dewormer concurrently combined with a refugia program. By deworming only the affected animals (targeted treatment), the other animals serve as a source of refugia.

In September in the Southeastern U.S., a two-year-old Angus heifer died the previous night. She had a three-week history of poor appetite, unthriftiness, and diarrhea. Several other younger cows in this herd also look unthrifty and have diarrhea. The cattle are vaccinated yearly and dewormed twice a year with pour-on moxidectin. Necropsy reveals that the abomasum is edematous and covered in small umbilicated nodules, 1-2 mm in diameter (cobblestone appearance). What is the most correct action to take next? A - Provide loose trace mineral salt to the entire group B - Vaccinate against Clostridium perfringens C and D with a multivalent bacterin C - Provide free-choice monensin and thiamine-supplemented feed D - Treat affected cows with 2 classes of dewormer E - Medicate for liver flukes

Report outbreak to regulatory authorities This history and the necropsy findings are consistent with viscerotropic velogenic Newcastle disease (VVND), which is reportable. The appropriate regulatory authorities should be contacted ASAP. Gross lesions such as the petechiae seen here in the proventricular mucosa are not usually seen with low-virulence Newcastle disease (loNDV). The clinical history is also suggestive of VVND. The acute form of NDV must be differentiated from highly pathogenic avian influenza by isolation of a hemagglutinating virus identified by inhibition with Newcastle disease antiserum. NDV occurs worldwide and chickens usually present with acute respiratory disease. Occasionally diarrhea, neurological problems, or depression predominate. Virulence varies between the VVND and loNDV forms. Vaccines against NDV decrease clinical signs and death. Infected poultry and depopulated. Sometimes the temperature of the poultry house is increased to decrease morbidity and mortality in flocks with suspected avian influenza. Amprolium is a treatment for coccidiosis.

Many hens in a large backyard flock recently became ill after the owner purchased four new chickens. The sick hens are gasping, coughing and sneezing. They have poor appetite and act depressed. Some affected birds have watery green diarrhea and swelling of the head and neck. A few have paralyzed legs and wings, twisted necks, are circling, and have tremors or clonic spasms. Birds are not laying well and some eggs are misshapen with watery albumen. Many of the sickest birds have died. Necropsy of a dead chicken shows petechial hemorrhages on the mucosal surface of the proventriculus and gizzard (left). What should be done next to address this problem? A - Treat all affected chickens with amprolium B - Cull affected birds and vaccinate the remainder C - Disinfect housing with phenolic compounds, barrier precautions for staff D - Report outbreak to regulatory authorities E - Increase poultry house temperatures to reduce morbidity

Pour-on eprinomectin Treat his chorioptic mange (leg mange) with pour-on eprinomectin or moxidectin (both approved for lactating dairy cattle). Pour-on doramectin and ivermectin as well as eprinomectin and moxidectin are approved for this use in beef cattle. Isolation is not practical because of the contagious nature of this disease. Assume all animals in the group are infested and treat accordingly. Classic chorioptic mange is seen in winter in the northeastern US. Cattle present with crusty, pruritic skin disease around the perineum and caudal upper thich. On skin scrape, look for long legs and short unsegmented pedicles in chorioptes. (in contrast to the short legs and long unsegmented pedicles of sarcoptes)

On a cold winter day in the Northeastern U.S., a lactating dairy cow is examined for pruritus and crusts around the perineum. A skin scrape shows some kind of mite with long legs and short unsegmented pedicles. What should be done next? A - Slaughter B - Pour-on eprinomectin C - Amitraz SQ D - Quarantine animal E - Topical furazolidone

Screen broodstock with qPCR and cull positives Renibacterium salmoninarum (bacterial kidney disease - BKD) is best prevented by screening broodstock with qPCR and culling any positives to prevent vertical transmission. Another option is to use ELISA to screen the broodstock and cull the eggs from positives. R. salmoninarum is one of relatively few gram-positive organisms that causes disease in fish and it is economically important. Fish appear lethargic with darkened skin and sometimes exophthalmia. There may be grayish granulomatous nodules on the kidneys. It is transmitted both horizontally and vertically and disease is more common in younger fish. Bacterial kidney disease is most common is both farmed and wild salmonids, but also occurs in other fish species. Purchase BKD/Renibacterium-free stock to avoid introducing this into the system. Previously it was recommended to treat females with erythromycin before spawning. However, this is not FDA-approved for treatment in farmed salmonids (there is an INAD available but not best use of antimicrobials and is difficult to get the drug) and survivors remain carriers.

Some farmed salmon appear lethargic with darkened skin and exophthalmia. Necropsy of one that died reveals vent hemorrhages and grayish granulomatous nodules on the kidneys. Gram stain from the kidney lesion reveals the intracellular bacteria shown below. How could this condition have been prevented? A - Test incoming fish with ELISA and use formalin for any positives B - Increase water temp in tank/pond to 52-63 ºF (11-17°C); Supplement diet with vitamin C C - Treat females with erythromycin before spawning D - Oxytetracycline in water; Cull fish older than 2 years E - Vaccinate all new arrivals

Papillomatosis These are warts, or papillomas, which is related to infection with equine papilloma virus. Equine papillomatosis occurs most frequently in young horses and are most commonly found on the inner surfaces of the pinna, muzzle, distal limbs, and genitalia. They are contagious and since they usually resolve spontaneous, treatment is rarely pursued.

This three-year-old horse is presented for evaluation of this dry, horny, wart-like mass on the distal forelimb. This horse also has smaller wart-like lesions on its muzzle. The other young horses in the herd have similar lesions. Which one of the following is the most likely diagnosis? A - Melanomatosis B - Papillomatosis C - Dermatophilosis D - Pediculosis E - Trombiculosis

Flehmen This is flehmen. This is a behavior seen in the following circumstances: normal stallion response to pheromones, normal response of all horses to new smells, sign of colic, or associated with granulosa cell tumors in mares.

What behavior is the mare displaying in the image below? A - Cribbing B - Aggression C - Windsucking D - Lignophagia E - Flehmen

Stars

What do the blue arrows indicate in the lower incisors of this horse? A - Stars B - Equine odontoclastic tooth resorption and hypercementosis (EOTRH) C - Cups D - Galvanyne's groove E - Cavities

Interruption of inhibitory neuron input from lumbar spinal cord Schiff-Sherrington posture occurs due to interruption of inhibitory neuron input from the lumbar spinal cords. The posture is characterized by rigid forelimb extension with normal function, with concurrent hindlimb paresis and paralysis. Trauma to the spinal cord between T3-L3 results in inhibitory pathway interruption. Neurons caudal to the cervical intumescence are released causing excessive extensor tone in the forelimbs. This posture is not associated with prognosis of the underlying disease. Think of Schiff-sherrington posture when you see a combination of forelimb extensor rigidity and a hind limb flaccid paralysis in an animal with spinal trauma, like being hit by a care or acute intervertebral disc disease.

What is the neuroanatomic cause of extensor rigidity in the forelimbs of a patient with Schiff-Sherrington posture? A - Polyradiculoneuritis of peripheral nerve sheaths at cervical intumescence B - Damage to thoracic spinocerebellar tracts in superficial white matter C - Interruption of inhibitory neuron input from lumbar spinal cord D - Brachial plexus trauma E - Subdural white matter compression at C6-T3

Anaplasma This is a tricky question to help you remember that two diseases formerly classified as Ehrlichia have no been reclassified. The causative organism of equine granulocytic anaplasmosis was originally classified as Ehrlichia equi, but is now called Anaplasma phagocytophilum due to DNA sequencing studies. Do not confuse equin anaplasmosis, a necrotizing vasculitis, with bovine anaplasmosis, which primarily causes an anemia with icterus and fever. Another name change occurred with the causative organisms of Potomac horse fever, from Ehrlichia risticii to Neorickettsia ristcii. PHF presents as a febrile colitis/diarrhea, with laminitis 3-5 days after diarrhea in horses of all ages: big rule out is salmonella (think septicemia/fever + diarrhea).

What kind of organism causes equine granulocytic anaplasmosis? A - Spirochete B - Ehrlichia C - Protozoa D - Chlamydia E - Anaplasma

Stomach The stomach is markedly enlarged and filled with fluid and gas. There is mineral opacity material with a sharp edge in the body and pylorus. There are small mineral opacities (gravel sign) in the dependent portion of the stomach. The enlarged stomach is causing a mass effect and displacing the other organs caudally, however peritoneal detail is good. The radiographs show evidence of pyloric outflow obstruction such as a mass, stricture, polyp, or granuloma.

Where, anatomically, is the predominant abnormality located in these radiographs from an 11-year-old English sheepdog? A - Liver B - Pancreas C - Spleen D - Stomach E - Right kidney

Palmar digital Palmar digital nerve block (also called a PD or hell block) would anesthetize at least the palmar third of the foot, including the navicular bone. Historically, the PD block was considered to anesthetize the palmar/planter 1/3 of the foot and sole. However, it has been shown that is desensitizes 70-80% of the foot in most horses. All of the other nerve blocks can desensitizes the navicular bone too, but would not be as specific as a palmar digital nerve block because they also desensitizes a larger region. The abaxial sesamoidean nerve block would anesthetize the entire food and much of the pastern. Sometimes an ASNB is needed to completely eliminate or diagnose navicular lameness, but it is less specific than the PD block. A low four point nerve block would desensitize the fetlock and areas distal to it. The median and ulnar nerve block would anesthetize the carpus and areas distal to it. The tibial and peroneal nerve block would anesthetize the tarsus and areas distal to it.

Which nerve block would most specifically relieve lameness resulting from fracture of the navicular bone? A - Tibial and peroneal B - Median and ulnar C - Low four-point D - Abaxial sesamoidean E - Palmar digital

Dystocia, overfeeding in dry period, Ca-P imbalance in feed Cow are predisposed the post-parturient metritis by many things, including: Dystocia, overfeeding in dry period, Ca-P imbalance in feed Cows are also predisposed to metritis by Retained placenta, contaminated calving environment, abortion, malnutrition In cattle, the causative bacterial organisms isolated most often are Trueperella pyogenes alone or with Fusobacterium necrophorum or other gram-negative anaerobes. Specific diseases associated with bovine metritis or endometritis include brucellosis, leptospirosis, trichomoniasis, and bovine campylobacter.

Which one of the following sets of conditions predispose a cow to metritis? A - Strep. agalactiae mastitis, retained placenta, laminitis B - Contaminated calving environment, abortion, hypomagnesemia C - Dystocia, overfeeding in dry period, Ca-P imbalance in feed D - Agalactia, milk fever, bovine vibriosis E - Milk fever, malnutrition, excess dietary zinc

Quarantine the horse; take temps on all other horses The horse should be quarantined immediately and all exposed horses monitored closely with twice-daily evaluation of temperature. This presentation is consistent with strangles, a highly contagious respiratory disease. You can confirm the diagnosis with PCR/culture of a nasal swab or swab of drainage from an abscessed lymph node. Strangles (Streptococcus equi sbsp equi infection) infects the lymph nodes of the upper respiratory tract and the ensuing lymphadenopathy can strangle the horse as seen in this case. It first causes fever, followed 24-48 hours later by mucopurulent nasal discharge and then lymph node enlargement and abscessation. As that disease progresses, the affected lymph nodes enlarge and eventually rupture and drain purulent exudate. If the retropharyngeal lymph nodes are affected, they can rupture and drain into the guttural pouches, setting up a carrier state. Most cases of strangles response to supportive care - judicious use of NSAIDs, facilitating abscesses maturation, flushing the abscesses once open/draining. Use of antibiotics depends on severity of clinical signs and the number/ages of horses affected.

A couple of days ago a client brought an 18-month-old filly home from a ?kill pen? (was headed for Mexico for slaughter). She just sent you photos of the horse showing thick bilateral mucopurulent nasal discharge and a swollen throat latch. She says that the horse's breathing is stertorous and loud. You are on your way to see the horse, thinking you will need to do an emergency tracheotomy first thing. What are additional preliminary recommendations based on the top differential? A - Pass a nasogastric tube to treat esophageal obstruction B - Refer for laser ablation treatment for guttural pouch tympany C - Start antimicrobials; limit iodine intake and measure thyroid levels for goiter D - Will need to biopsy the mass if owner wants to proceed with chemotherapy E - Quarantine the horse; take temps on all other horses

Upper left You are looking at an iliac fracture in the UPPER LEFT quadrant. Generally, when you assess pelvic rads after trauma, you are looking for pelvic fractures, femoral fractures (head and neck), or hip dislocation/luxation (usually cranio-dorsal). Sometimes, the priorities of triage require you to accept less-than-optimal radiograph positioning in the service of rapid assessment. The visible space evident at the sacroiliac junction on the upper right is not a subluxation, it is simply poor positioning - a common hazard when taking radiographs on an awake dog.

A dog hit by a car in the last three hours is presented recumbent and traumatized. Plain radiographs of the awake dog are shown below. Positioning is not perfect because the dog is in pain, but if there is a problem evident in the radiograph, in which quadrant does it lie? A - Upper left B - Lower left C - Upper right D - Lower right E - Looks normal

Normal age related change These are normal age related changes. Rate normally live 2-3 years, thought some may live to 5. With age, the hair coat of white rats normally develop yellowing fur. They may also show a brownish, granular sebaceous secretion at the base of affected hair shafts which an owner may mistake for a skin parasite.

A three-year-old white rat is presented with yellowing fur and a brownish, granular sebaceous secretion at the base of affected hair shafts. What should the owner be told? Normal age-related change Probable ringworm, resolves spontaneously with sunlight exposure Moisture-related dermatitis, cage needs more frequent cleaning Staph dermatitis, responsive to antiseptic shampoo Common manifestation of chorioptic mange

Foreign Body This dog has a chronic linear foreign body with multiple intussusceptions. The foreign material in the stomach and duodenum, gravel sign, and plication suggest a linear foreign body and chronic partial obstruction. Dissented bowel loops with a convex soft tissue appearance could indicate intussusception, however, ultrasound is better imagine modality to confirm this. Chronic partially obstructing linear foreign bodies can cause intussusception as the peristaltic motion of the intestines attempts to move the foreign material forward. Decreased serosal detail is likely in part due to poor body condition although the presence of peritoneal effusion is also likely.

A young adult terrier cross dog is presented for inability to gain weight, vomiting, and poor appetite. The owner had adopted the dog from a shelter several days earlier after he had been found in an emaciated state. On physical examination, he had poor body condition (3/9) and abdominal palpation revealed a tense abdomen with gassy enlarged loops of bowel and a firm linear structure. Abdominal radiographs are shown below. Which one of the following choices is the most likely diagnosis? A - Foreign body B - Normal abdomen for age C - Retroperitoneal mass D - Microhepatica E - Megacolon

Ultrasonography Use ultrasonography to identify testicular torsion in dogs with acute onset testicular swelling and pain. Other differentials include a scrotal hernia and epididymitis (usually bilateral). Tx with castration to remove the affected testicle. In the image the testicle on the right shows hemorrhage and infarction consistent with torsion.

An four-year-old male whippet is presented with an acute onset of swelling and pain in one of his testicles. A post-op image is shown below. Which test might have helped to diagnose the cause of the swelling before surgery? A - Lateral radiography B - Positive contrast cystography C - Nuclear scintigraphy scan D - Magnetic resonance imaging E - Ultrasonography

Acinar cell atrophy This dog has exocrine pancreatic insufficiency due to pancreatic acinar atrophy. PAA is the most common cause of EPI in GSD and is considered hereditary. It is also seen in rough-coated collies and Eurasians. In cats and older dogs, EPI is most commonly caused by chronic pancreatitis. Serum TLI is the best test for EPI in dogs and cats. A significantly low fasting TLI (<2.5) is diagnostic for EPI. in EPI there is impaired synthesis and release of digestive enzymes by pancreatic acinar cells, resulting in maldigestion, weight loss/failure to gain weight, diarrhea, and polyphagia. Concurrent SI dysbiosis and cobalamin deficiency are common. Treat with oral enzyme replacement and cobalamin supplementation if deficient. Patients with EPI require life-long enzyme replacement. Prognosis is good with appropriate treatment.

An eight-month-old male intact German shepherd dog is presented for poor weight gain, chronic diarrhea, and polyphagia. Fasting serum trypsin-like immunoreactivity (TLI) measurement is <2.0 µgL (Normal = 5.7-45.2 µg/L). What is the most likely cause of this dog's clinical signs? A - Acute pancreatitis B - Inflammatory bowel disease C - Acinar cell atrophy D - Pituitary dwarfism E - Small intestinal bacterial overgrowth

3.5-4 years 4th incisors are actually canine teeth. Cattle age is not always given in a history and estimating age during physical exam helps narrow differentials. References for times of eruption vary by about six months because the times of tooth eruption vary among individual animals and breeds. Full eruption of a new tooth may take another six months. Remember, ruminants don't have upper incisors, but instead, have a dental pad.

At what age do a cow's permanent 4th incisors begin to erupt in the mandible? 3 years 5-6 years 2 to 2.5 years 3.5 to 4 years 7-8 years

Camelids Camelids (llamas, camels, alpacas) normally have ellipsoid RBCs which lack central pallor. Birds and reptiles have nucleated, elliptical RBCs.

From which animal was this blood sample collected? A - Camelids B - Psittacines C - Ferrets D - Reptiles E - Chinchillas

Classical swine fever Virology is required to confirm and differentiate from other causes of febrile hemorrhagic diseases. The best tissues to submit are tonsils, maxillary or submandibular lymph nodes, mesenteric lymph nodes, spleen, ileum, and kidney. Glaesserella parasuis is an acute bacterial infection, characterized by different combinations of meningoencephalitis, polyserositis and polyarthritis and can contribute to bacterial pneumonia.

Over the past two weeks, several pigs in a herd have been febrile and depressed. Many were constipated, then had diarrhea. A few were incoordinated and one had seizures. A few have died. Necropsy revealed petechial hemorrhages on the kidneys and larynx, and a hemorrhagic urinary bladder. Of the following choices, which one is most consistent with a presumptive diagnosis? Classical swine fever Erysipelas Glasser's disease Streptococcus suis infection Swine dysentery

Laminitis In horses, retained placenta and septic metritis can lead to acute laminitis. There are four main types of laminitis: 1. sepsis/SIRs-related (following septic metritis, pleuropneumonia, colitis, grain overload) 2. Endocrinopathic (insulin dysregulation, equine metabolic syndrome, or equine Cushing's, lush spring grasses) 3. Traumatic (road founder) 4. Support limb (severe lameness on one limb requiring over-dependency on opposite leg)

Retained placenta and metritis can predispose to which one of the following secondary conditions in horses? A - Cystic endometriosis B - Colic C - Postpartum dysgalactia syndrome D - Laminitis E - Contagious equine metritis

Bovine viral diarrhea virus (BVD) In utero infection with BVD is a common cause of cerebellar hypoplasia in cattle. Other causes are bluetongue, Akabane, or wesselsbron viruses. Caudal occipital malformation syndrome is a disorder seen mainly in Cavalier King Charles Spaniels. The malformed occipital bone causes crowding at the caudal fossa and cerebellar herniation at the foramen magnum.

The calf shown below was presented with ataxia, intention tremors, and hypermetria; all signs were present since birth. The second image shows a normal bovine calf brain on the left and the brain from this calf on the right. Which one of the following choices is the most likely cause of the disorder in this calf? A - Bovine viral diarrhea virus B - Organophosphate toxicity C - Border disease D - Caudal occipital malformation syndrome E - Copper deficiency

Pemphigus foliaceus, pemphigus vulgaris, and bullous pemphigoid are generally considered type II reactions. Antibody binds antigen on a cell, then the antibody-antigen complex activates complement, causing cell lysis. In general, think of rare, autoimmune skin diseases characterized by varying presentations of ulcerations, crusting, pustules, vesicles. FYI - delayed hypersensitivity is the same as type IV reaction

The image below shows pemphigus foliaceus on a dog's abdomen. Pemphigus foliaceus, pemphigus vulgaris, and bullous pemphigoid are examples of which type of immune-mediated disease? A - Type III - antigen-antibody complexes deposited in tissues B - Type I - immediate IGE-mediated hypersensitivity C - Delayed hypersensitivity D - Type II - antibody binds cell antigen and activates complement E - Type IV - antigen triggers cell-mediated cytokine release, activates macrophages and production of cytotoxic T cells

Diazepam, lidocaine drip Treat the ventricular tachycardia with a lidocaine drip and address hyperactivity/seizures with diazepam (valium) or methocarbamol. If response to diazepam is inadequate, consider barbiturates like phenobarbital (given SLOWLY). This is classic presentation of severe chocolate toxicity, due to toxic methylxanthine alkaloids (theobromine, theophylline, caffeine). Common around Halloween, Christmas, and Valentine's Day holidays, see EXCITEMENT, SEIZURES, ARRHYTHMIAS. The seizure and ECG which shows ventricular tachycardia (V-tach) are key pieces of information here: Most chocolate toxicities are milder - hyperactivity, vomiting, urinating, diarrhea. Decontamination with emesis (induced with apomorphine) can be helpful up to eight hours after exposure in asymptomatic dogs. If animal is sedated because of seizures consider gastric lavage. If vomiting is controlled, give activated charcoal ONLY IF A LETHAL DOSE OF CHOCOLATE WAS INGESTED. Chocolate will increase the risk of hypernatremia from activated charcoal, so the benefit must outweigh the risk. If activated charcoal is given, monitor for hypernatremia and associated signs (tremor, ataxia, seizures). Rx arrhythmias depending on type - usually a tachycardia; give lidocaine for V-tach or beta-blockers (propranolol/esmolol) for supraventricular tachycardias. Chocolate toxicity from most toxic to least toxic: Cacao beans and baking chocolate worse than semisweet worse than milk chocolate. 49 grams of baking chocolate (one 2-oz bar) can kill a 7kg dog. It would take 420 grams of milk chocolate (about eight 2-oz bars) bars to kill a 7kg dog.

The night after Valentine's day, a male neutered Schnauzer is presented with a combination of hyperactivity, urinary accidents, and diarrhea. On physical exam there is muscle rigidity and chocolate wrappers visible in the diarrhea. The dog has a seizure on the exam table. An ECG is shown below. Vitals: T = 103.2 HR = 184 bpm RR = 40 bpm Which one of the following choices is the best treatment plan? A - Phenobarbital, glycopyrrolate B - Diazepam, lidocaine drip C - Ipecac, theophylline, digoxin D - Activated charcoal, sodium sulfate, methocarbamol IV E - Apomorphine, gastric lavage, Na-bicarbonate IV

Pour-on moxidectin These are the larvae of Hypoderma bovis or H lineatum (common and northern cattle grub respectively0 and pour-on moxidectin or eprinomectin are approved for use in both lactating dairy and beef cattle. Other products are approved for non-lactating and beef cattle. Withdrawal times must be followed. Organophosphates are not approved for this use in cattle in the US. While the grubs can be manually expulsed, care must be taken to prevent rupture of the grubs. Crushed larvae release antigens leading to a systemic anaphylactic reaction in cattle. These pests are less common now since macrocyclic lactones gained widespread use in the last few decades. The adult lies (gad/heel flies) lay eggs on the lower limbs of the cattle over the summer. the larvae penetrate the skin and migrate to their respective preferred locations. By early winter, the larvae arrive in the subcutaneous tissues of the back and make breathing pores. This cyst/warble stages lasts 4-8 weeks, culminating in 3rd stage larvae dropping to the ground and completing the life cycle. Clinical presentation of the larval infections range from localized tissue trauma around the warbles/cysts to periost

These larvae shown below have been removed from the topline of beef cattle in the early winter. Which one of the following treatments is the most appropriate in both lactating dairy and beef cattle? A - Organophosphate spray B - None. These are self-limiting C - Chloramphenicol D - Rapid hand expulsion/squeezing E - Pour-on moxidectin

It is an intact male. This is the penis of an intact male cat with backward-projecting cornified spines on the glans due to the presence of circulating androgens. Penile spines are absent in neutered males. The presence of spines in a previously neutered cat suggests cryptorchidism; spines should disappear within six weeks of castration.

What does this finding mean in a cat? A - Nasopharyngeal polyp, cat will be fine after surgical removal B - Cuterebra infestation C - It is an intact male D - Lingual polyp, can remove, but recurrence is common E - Female in estrus

Clostridioides difficile A history of recent antimicrobial therapy is common in cases of Clostridioides difficile associated diarrhea. Adult horses exposed to erythromycin are particularly at risk for C difficile enterocolitis. Clostridium novyi is the cause of infectious necrotic hepatitis, which is primarily seen in sheep, but can also be seen in cattle, hogs, and horses. Lawsonia intracellularis is the cause of proliferative enteropathy, resulting in diarrhea and hypoproteinemia in foals and swine. Rhodococcus equi is a notable cause of pneumonia in older foals characterized by pulmonary abscessation as well as some extra-pulmonary manifestations. Escherichia coli can be a cause of septicemia and diarrhea in foals and calves.

When a foal is being treated with erythromycin (for Rhodococcus equi, for example), the mare is at risk of developing enterocolitis due to which one of the following organisms? A - Lawsonia intracellularis (a.k.a. proliferative enteropathy) B - Escherichia coli C - Clostridium novyi D - Clostridioides difficile (formerly Clostridium difficile) E - Rhodococcus equi

1 hour Dystocia in the sow is present when one hour or more has passed between piglets. Intervention should be swift to prevent death of piglets from anoxia. In sows, dystocia is often due to uterine inertia, similar to other animals that produce multiple offspring. Other causes include fetal malposition, fetopelvic disproportion, obstruction of the pelvic canal, deviation of the uterus, and excitement in the dam. Medial therapy can be given with oxytocin and/or calcium only when an obstructive dystocia is not present.

When a sow is giving birth, what is the shortest time interval between passage of piglets that indicates dystocia? A - 15 minutes B - 4 hours C - 1/2 hour D - 1 hour E - 2 hours

Mediastinum This cat has a cranial mediastinal mass. There is a small volume pleural effusion causing widened fissure lines and retraction of the lung lobes. There is focal widening of the cranial mediastinum on the ventrodorsal projection, which is apparent as a soft tissue opacity flattening the margin of the cranial lung lobe on the lateral projection. The trachea is mildly elevated and narrowed in this area. The most common caused of a cranial mediastinal mass in cats are thymoma and thymic lymphoma. This cat was diagnosed with lymphoma. Rule out FeLV in cats with this condition.

Where is the location of the main lesion in this cat? A - Rib and body wall B - Trachea and mainstem bronchi C - Mediastinum D - Lung parenchyma E - Pulmonary vessels

Remove horses from affected pastures to help prevent mare reproductive loss syndrome (MRLS) The exact pathogenesis of MRLS is unknown, but exposure to ingestion of hirsute or hairy eastern tent caterpillar appears to be a risk factor. Early and late-term abortions have been reproduced by oral administration of whole eastern tent caterpillars (Malacosoma americanum) or their exoskeletons. Eastern tent caterpillars were associate with a large outbreak in Kentucky in 2001-2022 where over 3000 pregnant mares aborted. Other episodes have been seen in Floria and Australia with exposure to other animals of hirsute caterpillars. The most current theory - damage to the oral mucosa or intestinal lining by the setae (spines) of the caterpillars introduces bacteria that causes bacteremia and subsequent infection of the fetus and/or placenta. Early embryonic loss, abortion, stillbirths, weak foals, and infected foals are seen. Swollen umbilical cords are also often seen in aborted fetuses. Feeding hay and removal of wild cheery trees, a major food source for the caterpillars, is also beneficial in reducing the incidence of MRLS abortion.

Which one of the following management recommendations is most helpful to reduce the incidence of mare reproductive loss syndrome? A - Vaccinate mares and stallions against equine coital exanthema B - Cut down all the black walnut trees on premises C - Spray paddocks to kill Habronema muscae larvae D - Remove the horses from affected pastures E - Progesterone supplementation between four to six months of pregnancy

Buy only vaccinated chicks. This chicken is likely to have ocular manifestations or Marek disease. There is no treatment, but the problem is preventable when chicks are vaccinated at hatching or before 2 weeks of age. Diffuse infiltration of mononuclear cells can turn the irises of affected chickens from a normal yellow color to a pale tan to gray color ("gray eye"). Cellular infiltration can also occur in many other tissues (skin, nerves viscera), and there may be palpable tumors in the muscle and skin. Marek disease is common in commercial poultry. Diagnosis is usually based on enlarged nerves and lymphoid tumors in the viscera. Lymphoid leukosis is a key differential. Absence of bursal tumors helps distinguish Marek disease from leukosis. But if you find bursal tumors, Marek disease is still on the DDX list pending other tests, like immunochemistry. Chickens as young as 3 weeks can show Marek disease, but lymphoid leukosis is typically seen in chickens more than 14 weeks old.

A 12-week-old chicken from a young backyard flock is presented for evaluation because of weight loss, decreased appetite, and diarrhea. The irises in the affected bird are lighter than normal with irregular pupillary margins, and the hen does not have a normal pupillary light reflex. Several other young birds appear to be growing poorly compared to the rest of the flock. The image below is an example of what the affected bird's eye looks like, vs. a normal eye on the left. Based on the presumptive diagnosis, how can this problem be prevented going forward? A - Cull birds from the same genetic line B - Isolate unaffected animals; Breed from survivors of disease C - Collect pharyngeal swabs from affected birds; Antibiotic Tx for entire flock based on culture and sensitivity D - Use only mycoplasma-resistant bloodlines for new additions to flock E - Buy only vaccinated chicks

Left cranial lung lobe has alveolar infiltrates This dog has pneumonia of the left cranial lung lobe. There are diffuse alveolar infiltrates within the left cranial lung lobe with air bronchograms. You should cancel the anesthetized procedure and wake up the patient because she is oxygenating poorly. Recheck radiographs are recommended once she is awake to rule out any component of atelectasis due to recumbency and sedation. There is an incidental gastric foreign body - rounded metal opacity within the stomach that is in close associated with additional irregular metal opacity The dog dose has spondylosis deformans, which is very common and incidental radiographic finding in older dogs. Although these changes are present, they are not the most clinically important findings on these radiographs.

A 12-year-old female spayed Labrador Retriever dog is presented for dental cleaning. After she is induced under general anesthesia, her pulse oximetry drops below normal at 91%. Thoracic radiographs show the following changes (see images below). What is the most clinically significant finding evident on thoracic radiographs from this dog? A - Osteochondritis dissecans is evident in the left shoulder B - Heart is globoid in appearance C - There is a gastric foreign body D - Left cranial lung lobe has alveolar infiltrates E - Spondylosis deformans is present

Thoracocentesis and chest tube placement This puppy has a pneumothorax and needs immediate thoracocentesis and chest tube placement. The left hemithorax appears gas distended with marked rightward mediastinal shift. A large, loculated area of subcutaneous emphysema is seen over the caudal dorsolateral thorax; however, a communication with the thoracic cavity of the external body wall cannot be identified. A triangular wedge of soft tissue opacity is seen in the mid left thorax and is consistent with collapsed left lung. A smaller, poorly demarcated area of increased soft tissue opacity is also seen superimposed over the 10th left rid adjacent to the body wall at the level of the most sever subcutaneous emphysema. There is mild pleural effusion in the right hemithorax. The right pulmonary parenchyma is difficult to evaluate, but appears to be within normal limits. The musculoskeletal structures appear within normal limits. This is sever left sided tension pneumothorax with collapse of the left lung lobes.

A 13-week-old puppy is presented for trauma after being attacked by another dog. After stabilization, a series of radiographs show various issues (see below). Which one of the following choices is the best next step to do for this patient? A - Computerized tomography B - Place an intravenous catheter and start IV fluids C - Thoracocentesis and chest tube D - Suture lacerations and insert drains E - Aspirate subcutaneous emphysema and apply snug body wrap

High ringbone This is an example of high ringbone (i.e. DJD of proximal interphalangeal joint). Very common in horses. Can be due to chronic wear and tear, overuse, or secondary to a traumatic episode. Osselets are traumatic arthritides of the metacarpophalangeal joints. Pedal osteitis is demineralization of the solar margin of the distal phalanx, usually due to inflammation. Bone spavin is osteoarthritis of the distal intertarsal, tarsometatarsal, and less commonly, the proximal intertarsal joints.

A 15-year-old Quarter horse mare is presented for right front lameness grade 3/5 on the AAEP lameness scale (lameness consistently visible at the trot). She becomes sound after an abaxial nerve block. Radiographs of the distal limb look are shown below. What is the common name for the top differential diagnosis? A - Pedal osteitis B - Spavin C - Chronic proliferative synovitis D - High ringbone E - Exostosis of the second metacarpal

Measure resting ACTH and insulin These hooves show evidence of chronic laminitis. One of the most common causes of chronic laminitis is equine metabolic syndrome (EMS), characterized by insulin dysregulation, regional or generalized adiposity, and laminitis Grazing is typically contraindicated in horses with EMS because the fructans in fresh grass exacerbate the insulin response and laminitis. Steroids also worsen insulin dysregulation and should be avoided in horses with EMS.

A 16-year-old retired Thoroughbred mare is presented for spring vaccinations. Her feet look like this (see image below). Which one of the following would be recommended? Check serum vitamin E and selenium levels Measure resting ACTH and insulin Recommend 24/7 pasture turnout with grazing Ask if she's being fed a diet with excessive wheat bran, leading to an imbalance in the calcium:phosphorus ratio Start on a course of systemic steroid therapy

Retained egg This bird has a retained egg. The lateral radiograph demonstrated a 4.5cm diameter, well-circumscribed, soft tissue opacity in the central abdominal area. It extended from the caudal hepatic border to the caudal pelvic region. Its dorsal margin was relatively distinct and did not silhouette with the kidneys. Evidence of calcification was not identified. Increased endosteal opacity was present in the pelvic extremities. The remained of the study was unremarkable except for mild decreased liver size.

A 21-year-old scarlet macaw is presented with polydipsia, watery droppings and inappetence. On examination, the bird was fluffed and depressed with minimal pectoral muscle atrophy. The crop was fluctuant. Which one of the following choices is the most likely diagnosis? A - Vitamin D toxicosis B - Retained egg C - Normal radiographs D - Air sacculitis E - Thymoma

Withhold food and water - give sedatives and oxytocin - recheck in 2 hours The most appropriate initial therapy for this systemically stable horse with an acute esophageal obstruction (choke) is to withhold food and water, give sedatives and oxytocin (and n-butylscopolammonium bromide) to relax the esophagus, and recheck in a couple hours. This is often successful in uncomplicated cases. If obstruction has not resolved within 2-4 hours, sedate to lower the head, pass a nasogastric tube to the level of the obstruction, and GENTLY lavage the esophagus with water. This is typically successful. In cases that do not resolve with this treatment, refer for ultrasound, esophageal endoscopy, and/or general anesthetic with esophageal lavage at a referral center. Obstructions lasting more than several hours require IV fluids and electrolytes, anti-inflammatories, and antimicrobials (for secondary aspiration pneumonia). Complications include aspiration pneumonia repeat obstruction, and esophageal stricture.

A 22-year-old Thoroughbred mare is presented for bilateral green nasal discharge, retching, coughing, and inappetence noted after feeding this evening. The owner recently switched the mare to a pelleted diet because of the mare?s poor dentition. Physical exam reveals bilateral green nasal discharge. The mucous membranes are pink and moist with a capillary refill time of 1.5 seconds. TPR - 100.6, 40 bpm, 12 bpm A slight swelling is notable in the proximal-left jugular groove just caudal to the vertical ramus of the mandible. Based on the most likely diagnosis, what initial therapy would be recommended? A - Pass nasogastric tube; give atropine; lavage the esophagus until the obstruction clears B - Ultrasound thorax, perform transtracheal wash for cytology and culture, begin broad-spectrum antimicrobials C - Cautious guttural pouch endoscopy with lavage for Streptococcus equi spp. equi PCR, quarantine horse until results are available D - Administer antimicrobials, refer horse immediately for endoscopy and lavage under general anesthesia E - Withhold food and water; give sedatives and oxytocin; recheck in 2 hours

Both mother and baby are safe The toxoplasmosis organism causes birth defects in a developing fetus if a mother is infected for the first time in her life while pregnant. Toxoplasmosis is not generally dangerous to immune-competent people and a positive IgG result suggests a previous infection. If a pregnant woman is IgM positive, confirmatory tests must be done.

A female veterinarian is pregnant and her physician tests her serologically for toxoplasmosis. IgM is negative. IgG is positive. What is the most appropriate interpretation? A - Mother at risk, baby is safe B - Both mother and baby are safe C - Both mother and baby at risk D - Mother is safe, baby at risk E - Need to re-check in 2 weeks for rising titers

Estrus This dog is in estrus. Look for >90% cornfield superficial cells (angular, sharp edges like cornflakes, tiny pyknotic nuclei or no nuclei and no neutrophils). In contrast, during diestrus vaginal smears have >50% nucleated, round-edged parabasal cells. Look for the reappearance of neutrophils. Full cornification usually coincides with receptivity. Use serum progesterone monitoring with vaginal cytology to predict the LH surge and help estimate ovulation. The initial progesterone rise in considered day 0, breed twice during days 3-6 for maximal breeding success.

A canine vaginal smear is shown below. At what stage of the estrous cycle is this dog? A - Diestrus B - Anestrus C - Proestrus D - Cannot tell from this slide E - Estrus

Remove via endoscopy Endoscopy would be the safest route of removal in this case. The cap is large enough to be lodged in the intestinal tract if allowed to pass. With endoscopy, you can get it out gently and quickly, with minimal anesthesia and no suturing. This is preferable to gastrotomy which would require incisions in both the abdominal wall and the stomach.

A client presents a 25-pound dog (11.4 kg) who was just observed swallowing a bottle cap. Which one of the following choices would be the safest approach for this dog? A - Remove via gastrotomy B - Administer silicon coated magnet to keep it in stomach until dissolution C - Remove via endoscopy D - Administer a proton pump inhibitor to stimulate gastric acid secretion E - Administer motility modifiers to encourage passage

Actinobacillus pleuropneumonia This is a classic scenario of an outbreak of pleuropneumonia, caused by Actinobacillus pleuropneumoniae. Primarily a growing pig disease, although adults can be affected as well, and sows can abort. Mortality is high if untreated. Survivors may experience lowered growth rates and have a persistent cough.

A commercial pig farm experiences an explosive outbreak of respiratory disease with high mortality, primarily in young pigs under six months of age. Affected pigs show severe respiratory distress, fever up to 107°F (41.5°C), anorexia, and reluctance to move. Some animals display open-mouth breathing with a blood-stained, frothy nasal and oral discharge. On necropsy, the lungs are bilaterally dark and swollen and ooze bloody fluid from the cut surface, as shown in the image below. Which one of the following choices is the most likely diagnosis? A - Fusobacterium necrophorum B - Actinobacillus pleuropneumoniae C - Haemophilus parahaemolyticus D - Mycoplasma hyopneumoniae E - Swine influenza

Ascites syndrome This is ascites in a chicken. Suddenly dead or cyanotic, panting chickens with abdomens distended by ascites fluid suggests ascites syndrome/waterbelly. Essentially a form of right ventricular heart failure secondary to pulmonary hypertension or more rarely hepatic failure. Because broilers affected with ascites syndrome frequently die on their backs, differential diagnosis includes flip-over disease. Another cardiomyopathy to remember is round heart disease (spontaneous cardiomyopathy), but that is a disease of turkeys. Dissecting aneurysm is another cause of sudden death, but animals display severe hemorrhage on necropsy.

A dozen young chickens at a broiler-raising facility are found dead on their backs and sides with no premonitory signs. Other birds appear cyanotic, with distended and reddened abdomens and congested blood vessels. Affected birds have an increased respiratory rate and reduced exercise tolerance. Necropsy of a dead chicken shows typical lesions below. What is the clinical diagnosis? A - Dissecting aneurysm B - Ascites syndrome C - Spontaneous cardiomyopathy D - Infectious bursal disease E - Flip over disease

Circovirus Porcine circovirus 2 (PCV2) is a non-enveloped DNA virus first described in Canada in 1991 that is ubiquitous today worldwide. It causes post-weaning multi-systemic wasting syndrome (PMWS) as it attacked the immune system. It is preventable with vaccination. The virus is also associated with porcine dermatitis and nephropathy syndrome caused by necrotizing vasculitis, which is more prominent in the skin, renal pelvis, mesentery, and spleen.

A finishing farm has pigs that are about 12 weeks old. The farmer is noticing that about 10% of the pigs are failing to thrive, have a yellow watery diarrhea, are becoming emaciated, and dying - even if treated with antibiotics. Affected pigs are pale to icteric. Examination reveals enlarged inguinal lymph nodes. What is the causative agent? A - Herpesvirus B - Manangle virus C - Parvovirus D - Paramyxovirus E - Circovirus

Circovirus Porcine circovirus 2 (PCV2) is a nonenveloped DNA virus first described in Canada in 1991 that is ubiquitous today worldwide. It causes post-weaning multi-systemic wasting syndrome (PMWS) as it attacks the immune system. It is preventable with vaccination. The virus is also associated with porcine dermatitis and nephropathy syndrome caused by necrotizing vasculitis, which is more prominent in the skin, renal pelvis, mesentery, and spleen.

A finishing farm has pigs that are about 12 weeks old. The farmer is noticing that about 10% of the pigs are failing to thrive, have a yellow watery diarrhea, are becoming emaciated, and dying - even if treated with antibiotics. Affected pigs are pale to icteric. Examination reveals enlarged inguinal lymph nodes. What is the causative agent? A - Parvovirus B - Paramyxovirus C - Manangle virus D - Herpesvirus E - Circovirus

Bacterial infection There are both intracellular and extracellular bacteria present in this cytology. The abscess should be drained and the animal placed on appropriate antibiotics. MCT are round cell tumors with basophilic granules that are smaller than bacteria.

A five-year-old boxer is presented for routine vaccinations. The owner points out a small, smooth mass on the right ear. Fine needle aspiration cytology is shown below. Which one of the following choices is the most likely diagnosis? A - Aural hematoma B - Degranulating mast cell tumor C - Histiocytoma D - Bacterial infection E - Lymphoma

Facial nerve Think of facial nerve (CN7) paralysis with a unilaterally droopy face. Remember, the facial nerve is MOTOR to the muscle of facial expression - causing dropped ear, lip, and eyelid - and innervates lacrimal and salivary glands. Loss of lacrimal innervation can lead to dry eye and exposure keratitis if dog loses ability to close eyelid from damage to facial nerve innervation of the orbicularis oculi muscle. Idiopathic in 75% of canine cases (25% of cats). Can also see these signs with middle ear damage (otitis media), facial nerve trauma (ear sx in dogs, halter buckle pressure in anesthetized horses) or neoplasia. Think more of a dropped jaw with trigeminal neuropathy (CN5) in dogs and horses.

A five-year-old female spayed Cocker Spaniel is presented with a drooping right ear and lip and drooling. There is ptosis OD (right eye) but the remainder of the physical exam is unremarkable. She is otherwise bright, alert, and responsive. What anatomic structure is damaged? A - Trigeminal nerve B - Facial nerve C - Left side inner ear D - Left side medulla, motor tract E - Right side cranial cervical ganglion

Pannus This is pannus, a bilateral, progressive, chronic superficial keratitis. DDX from other forms of keratitis by breed and progression. Pannus begins laterally at the limbus and extends from all sides to cover cornea. Common in GSDs also seen in Australian shepherds, border collies, greyhounds, huskies. Any keratitis (corneal inflammation) may also have ulceration. DDX corneal ulcers with fluorescein dye. Look for more corneal disruption melting with corneal ulcers.

A five-year-old female spayed German shepherd dog is presented with a bilateral eye problem which has been worsening over the past month, see the image below. What is the diagnosis? A - Chronic superficial corneal ulcer B - Anterior uveitis C - Pannus D - Descemetocele E - Interstitial keratitis

Exploratory laparotomy Ex lap is the most appropriate next step for this LFB. On the lateral rads, there is an area of the small intestine with very small gas bubbles where the walls of the SI appear undulating. In this case, the LFB was a bikini string. LFB is surgical emergency due to potential intestinal perforation and septic peritonitis. In severe cases, the sawing action of the LFB against the intestinal mucosa can cause extensive damage requiring extensive resection and anastomosis. Occasionally, the intestinal damage is too extensive and euthanasia is the best course. If a. definitive diagnosis is not evidence on radiographs, consider abdominal ultrasound to further evaluate the GI tract.

A five-year-old female spayed cat is presented with a two-day history of vomiting and anorexia. Physical exam reveals 5% dehydration and abdominal discomfort. Plain abdominal radiographs are shown below. Which choice is the most appropriate step to take next? A - Gastroduodenoscopy with biopsy B - Non-iodinated contrast series C - IV fluids and broad-spectrum antibiotics D - Exploratory laparotomy E - Specific feline pancreatic-lipase assay

Common opening of the pancreatic and common bile ducts into the duodenum. In cats, there is a common opening of the pancreatic and common bile ducts into the duodenum. It is thought that this may predispose them to ascending cholangitis and pancreatitis after vomiting associated with inflammatory bowel diseases, resulting in extrahepatic biliary obstruction. Other possible etiologies of EHBO include neoplasia, biliary structure, duodenal obstruction, diaphragmatic hernia, and parasitic infection. In all cases, there is a lack of bile entry into the intestinal tract, leading to decreased absorption of fat and fat soluble vitamins such as vitamin K, potentially resulting in coagulopathies. Except in transient cases that are related to acute pancreatitis, surgical intervention to relive the obstruction is required as well as appropriate supportive care, antimicrobial treatment, and vitamin K administration.

A five-year-old female spayed domestic short-haired cat is presented with chronic vomiting, anorexia, and weight loss. Physical examination reveals icterus, dehydration, and fever. Thickened small intestinal loops are appreciated with abdominal palpation, and serum biochemistry shows increased bilirubin and liver enzymes. The cat is diagnosed with extrahepatic biliary obstruction with accompanying cholangitis and inflammatory bowel disease. Which one of the following normal anatomic features may predispose cats to this triad of diseases? A - Narrowing of the duodenum distal to the entry of the common bile and pancreatic ducts B - Enlarged duodenal papilla in comparison to other species C - Decreased proportion of alpha-smooth muscle actin in muscle fibers of the wall of the gall bladder in cats D - Common opening of the pancreatic and common bile ducts into duodenum E - Decreased diameter and increased branching of the hepatic ducts in comparison to other species

Histomoniasis This is histomoniasis; the combination of characteristic "bull's-eye" lesions on liver and cecal changes are pathognomonic. Caused by protozoan H. meleagridis transmitted in eggs of cecal nematode Heterakis gallinarum. Expect a depression/diarrhea presentation. Expect to see more sudden death with necrotic enteritis caused by Clostridium perfringens. (Turkish towel). Signs of avian spirochetosis are highly variable, may be absent - see listlessness, shivering, increased thirst, green/yellow diarrhea with increased urates early on. Caused by a tick-borne Borrelia. Look for characteristic enlarged, mottled spleen with petechial. hemorrhages, similar to marble spleen disease of pheasants. Expect depression, bloody droppings, and substantial mortality with hemorrhagic enteritis of turkeys. (enlarged spleen, hemorrhagic intestines) Expect diarrheal presentation with coronaviral enteritis of turkeys, but NOT the characteristic cecal/liver lesions described on necropsy above. Good resource for poultry DZ - https://partnersah.vet.cornell.edu/avian-atlas/#/

A flock from a turkey farm is presented with a mysterious illness. Several dead birds are notable, mostly younger animals. Sick turkeys are listless, with drooping wings, unkempt feathers, yellow droppings. Sick older birds are emaciated. Necropsy shows a yellowish green, caseous exudate in the ceca, cecal ulcerations, and thickening of the cecal wall. A typical liver looks like the image below. What is the diagnosis? A - Necrotic enteritis B - Hemorrhagic enteritis of turkeys C - Avian spirochetosis D - Histomoniasis E - Coronaviral enteritis of turkeys

Bronchoscopy is the next best step. On radiographs of the thorax, there is an alveolar pulmonary pattern in the left cranial and right middle lung lobes, with patchy increased opacity in the remaining lobes. The trachea is narrowed at the thoracic inlet with an apparent linear intraluminal opacity. There is a dilation of the pharynx with air, and there is gas within the esophagus and GI tract. The tracheal narrowing and intraluminal opacity may indicate edema, mucus, or foreign material in this region There is secondary upper airway obstruction as indicated by the dilated pharynx and aerophagia. The alveolar pattern is due to bronchopneumonia.

A four-month-old Jack Russell terrier puppy was playing outdoors. Later that evening the owner noticed that the puppy was wheezing and coughing. Based on the radiographs, what would be the best step to take next? A - Barium swallow B - Trans-tracheal wash C - Dental prophylaxis D - Edrophonium challenge test E - Bronchoscopy

Cerebellar hypoplasia This is cerebellar hypoplasia, although cerebellar abiotrophy is also possible. This is a congenital, irreversible condition. Think of kittens with cerebellar hypoplasia due to in utero feline panleukopenia infection. MRI interpretation: Representative T1 and T2 weighted MRI images. The cerebellum is small with a resulting large collection of CSF in the vicinity. There are no other abnormalities of signal intensity.

A four-month-old female mixed breed puppy is presented with a one-month history of generalized wobbliness when walking and head-bobbing when attempting to eat. The signs have not progressed. What is the diagnosis? A - Cerebellar hypoplasia B - Distemper encephalitis C - Normal MRI D - Lead toxicosis E - Chiari malformation

Clarithromycin and rifampin Start this filly on clarithromycin and rifampin. In this case, a chest radiograph with consolidated nodular lung lesions and mediastinal lymphadenopathy is highly suggestive of Rhodococcus equi in a foal under five months of age. Polyarthritis, polysynovitis, and diarrhea can also be seen in affected foals. Combination treatment of rifampin combined with either erythromycin, clarithromycin, or azithromycin is appropriate to treat R equi pneumonia. However, in one retrospective study, the combination of clarithromycin and rifampin was shown to be the most effective of the three. Tilmicosin, while a macrolide, is not very active against R equi and swelling can occur at injection sites in foals; it is not recommended for treatment of R. equi pneumonia.

A four-month-old filly is presented with a four-day history of lethargy, heavy breathing, cough and decreased appetite. Vitals: 104.9 degrees, HR 72 bpm, RR 44 bpm On lung auscultation asymmetrically-distributed crackles and wheezes are audible. Some areas have no breath sounds and a dull resonance on thoracic percussion. A lateral chest radiograph shows consolidated nodular lung lesions and mediastinal lymphadenopathy. Pneumonia due to Rhodococcus equi infection is confirmed based on culture and polymerase chain reaction results on a transtracheal wash. Which one of following choices is the most appropriate treatment? A - Chloramphenicol and aminophylline B - Clarithromycin and rifampin C - Tilmicosin D - Procaine penicillin G and gentamicin E - Vancomycin and atropine (if bronchospasm is observed)

This is vena cava syndrome, surgery is best option This is vena cava syndrome/class IV heartworm disease and treatment of choice is surgical worm removal from the right atrium using fluoroscopy and long flexible alligator forceps. Do not administer an adulticide due to the risk of fatal compilations. The prognosis is grave is form removal is not possible Worm removal typically results in rapid resolution of clinical signs. After full recovery from surgery begin adulticide therapy with melarsomine.

A four-year old dog is presented with a history of sudden onset of collapse and respiratory distress. Physical exam shows pale mucous membranes, jugular distention and a jugular pulse. On an echocardiogram, a mass of worms is noted in the right atrium and caudal vena cava. What should the owner be told? Prognosis is fair, must stabilize first, then reassess The prognosis is hopeless and euthanasia is strongly encouraged Need to hospitalize, treat medically (Melarsomine 3 dose protocol) This is vena cava syndrome, surgery is best option Need to hospitalize, treat medically (Ivermectin/heparin protocol)

Granulomatous meningoencephalitis The lesion in this case localizes to the left cerebellum causing the left hypermetria and a right paradoxical head tilt. Granulomatous meningoencephalitis often affects this area. Another top differential would be cerebellar neoplasm.

A four-year-old Rat Terrier is presented with a three day history of progressive stumbling and falling. Physical exam reveals a right head tilt, left sided hypermetria, generalized ataxia and vertical nystagmus. Which one of the following choices is at the top of the differential list? Ascending and descending myelomalacia Granulomatous meningoencephalitis Fibrocartilagenous embolism Central pontine myelinolysis Amyotrophic lateralizing sclerosis

Granulomatous meningoencephalitis The lesion in this case localizes to the left cerebellum causing the left hypermetria and a right paradoxical head tilt. Granulomatous meningoencephalitis often affects this area. Another top differential would be cerebellar neoplasm.

A four-year-old Rat Terrier is presented with a three day history of progressive stumbling and falling. Physical exam reveals a right head tilt, left sided hypermetria, generalized ataxia and vertical nystagmus. Which one of the following choices is at the top of the differential list? Ascending and descending myelomalacia Granulomatous meningoencephalitis Fibrocartilagenous embolism Central pontine myelinolysis Amyotrophic lateralizing sclerosis

Treat entire group with an appropriate amtihelmintic Those not showing clinical signs may still be subclinically infected and benefit from treatment. Products used to treat lungworms also impact GI nematodes, so treating an entire group will impact GI nematode refugia and anthelmintic resistance. This should be factored into your treatment decisions. Lungworm infection (verminous pneumonia) is caused by Dictyocaulus viviparous. It is seen mostly in yearlings on pasture in the summer and fall. The primary clinical sign is coughing, Diagnosis is by finding larvae (not eggs) in feces or adult worms in the airways on necropsy. 4-Ipomeanol toxicity from moldy sweet potatoes or perilla mint can present similarly, but necropsy shows pulmonary edema and emphysema.

A group of yearling stocker cattle on summer pasture are presented for coughing in approximately 30% of the group. The owner has treated the coughing animals with antibiotics with no response. When the herd is gathered for examination the coughs worsen and some animals become dyspnic. Some of the cattle look thin and have rough haircoats. Necropsy of one animal shows the following. What is the appropriate action for this herd? A - Treat the coughing animals with an anthelmintic B - Call the state vets office to report a possible foreign animal disease C - Check for moldy sweet potatoes and Perilla mint D - Do nothing as this is a self-limiting problem E - Treat the entire group with an anthelmintic

Review infectious bovine rhinotracheitis (IBR) vaccination protocols Evaluation of the herd vaccination program is a top priority to prevent infectious bovine rhinotracheitis (IBR) with pustular vulvovaginitis from occurring again. Evaluating the herd biosecurity protocol is also important to prevent introduction of NEW cases. IBR is a common infection in cattle. Infected animals develop fibrinonecrotic plaques at the site of inoculation and, from these, the virus ascends to the regional ganglia ad becomes latent. During stressful episodes, recrudescence and shedding of latent virus occurs, perpetuating infection in herds. There are many vaccine products and protocols to help prevent outbreaks. There are several different types IBR virus with different tissue specificities. One type causes mucosal lesions and respiratory or vaginal infections. A second type primarily causes abortion, and a third causes fatal nonsuppurative encephalitis in calves. IBR is identifiable by fibrinonecrotic plaques on the mucosal epithelium, and may be confirmed by collected of nasal or ocular swabs for virus isolation, ELISA, or fluorescence antibody testing.

A herd of 25 Holstein heifers present with fever of 104-106°F, lethargy, and purulent nasal discharge. They also have loud, harsh coughs. Physical exam reveals crackles over the trachea, but not the lungs. Fibrinonecrotic plaques are randomly dispersed over the nasal and ocular mucosa, but there are no oral ulcers. Urine and feces appear normal. Several affected heifers have an arched back and are straining because of a fibrinonecrotic vaginitis. Which choice is the most important step to take next to evaluate and prevent this problem in the future? A - Test water for Leptospira serovars; Check feed for fecal contamination by dogs with Neospora caninum B - Check for elevated mycotoxin levels in feed, particularly zearalonone due to Fusarium spp. C - Review infectious bovine rhinotracheitis (IBR) vaccination protocols D - Perform vaginal cultures for genital Campylobacter fetus venerealis E - Evaluate breeding program; Consider 100% artificial insemination or use of Tritrichomonas foetus-negative bulls

Toxoplasma gondii T. gondii infection can cause severe abortion storms in sheep. Clinical signs pathognomonic of T. gondii are bright to dark red cotyledons with white foci of necrosis. The intercotyledonary areas of placenta are normal. With brucella and histophilus ovis, the abortion rate is much lower. Abortion caused by Listeria spp is associated with the feeding of silage.

A herd of ewes has had an abortion storm in which 80% of animals are affected. All stages of gestation are affected, and lambs that are born are very weak. On the placentas, cotyledons have white foci of necrosis. Which one of the following choices is the primary differential diagnosis? A - Listeria monocytogenes B - Chlamydiosis C - Histophilus ovis D - Brucella E - Toxoplasma gondii

Coagulopathy Coagulopathies are common in associated with crotalid envenomation. Additional concerns include thrombocytopenia, neurotoxicity, and cardiovascular toxicity. Temporary tracheostomy should be performed immediately in a horse in severe respiratory distress due to upper airway obstruction. The physical examination and further supportive care can be delayed until after an airway is obtained. Additional diagnostics include CBC/blood smear/serum biochemistry/coagulation profile. See hypofibrinogenemia, thrombocytopenia, prolonged prothombin time, and increased FDPs. Might see echinocytes in heparin or citrate samples, and severity of echinocytosis is associated with severity of envenomation. RX: IV fluids for hypotension, NSAIDs for analgesia, steroids are controversial, tetanus prophylaxis, antimicrobial prophylaxis, antivenin if owner can afford/it is available, and occasional blood transfusion. If you look closely at the picture of this horse, you can see he's receiving a blood transfusion.

A horse is presented in severe respiratory distress in southern California after a suspected snake bite. There is significant nasal swelling (see below), flared nostrils, abdominal movement with each breath, stridor, and tachypnea. Aside from local tissue damage and associated upper airway obstruction, what is another major complication of crotalid envenomation? A - Coagulopathy B - Chronic visual impairment C - Immune suppression D - Diarrhea E - Acute hepatic necrosis

Antigen-antibody complex (TYPE III) In type III reactions such as purpura hemorrhagic, antigen-antibody complexes are deposit on the endothelium, stimulating complement and a neutrophilic inflammatory response and vascular damage. The petechiae and distal limb edema in this horse are consistent with vasculitis. Classic type III diseases include: Glomerulonephririts Hypersensitivity pneumonitis (think moldy hay) Purpura hemorrhagic (think post-strangles infections, but possible with other infections as well) Anterior uveitis

A horse presents with distal limb edema in all 4 limbs (as seen below), petechiae of the mucous membranes, depression, and a low-grade fever of 101.9°F/38.8°C (N = 99-101°F, 37.2-38.3°C). Multiple horses in the stable recently suffered from an upper respiratory infection of undiagnosed etiology. Which type of immune reaction is most likely responsible for the vasculitis in this horse? A - None of these B - Cell-mediated immune reactions (type IV) C - Antibody-mediated cytotoxic reactions (type II) D - Antigen-antibody complex (type III) E - Immediate hypersensitivity and atopy (type I)

Coelom This owl has a coelomic mass. On survey radiographs, the right wing amputation is evident. The bird's body condition is thin. The heart appears mildly enlarged. There is a large, soft tissue opacity mass occupying the majority of the celom. The proventriculus is displaced ventrally, and the ventriculus is not clearly seen. 7mL 50% barium suspension was used to perform an upper GI exam. This revealed a ventrally displaced proventriculus, and the intestines wrapped peripherally around the mass. The brush border visible on the mucosal surface is normal. The intestinal tract appears to be patent with barium present in the cloaca at 30 min.

A long eared owl with a history of wing amputation is presented with anorexia, lethargy, and ataxia. Based on the images shown below, which one of the following structures has pathology? A - Ventriculus B - Cloaca C - Gizzard D - Proventriculus E - Coelom

Thoracic radiographs to evaluate for CHF This is classical presentation for dilated cardiomyopathy and the most appropriate next step is thoracic radiographs to evaluate for CHF. Echocardiogram can confirm the diagnosis. Prognosis is guarded - may live for 6mo-2yr with treatment, prognosis poor for dobies. Large and giant breed dogs at increased risk (irish wolfhounds, danes). Boxers can develop arrhythmogenic right ventricular cardiomyopathy, district from DCM. Treatment goal is to diminish overt clinical signs and control arrhythmias. Therapy may include diuretics, positive inotropes, angiotensin converting enzyme inhibitors, and antiarrhythmics. Dobermans have two genetic mutations that can lead to development of DCM - PDK4 and DCM2. Dogs affected with both at 30 times more likely to develop DCM.

A middle-aged, depressed, coughing, exercise-intolerant Doberman pinscher is presented. The dog has a rapid and irregular heart rate. Which initial diagnostic step is most appropriate? Ultrasound for pericardial effusion Thoracic radiographs to evaluate for congestive heart failure Abdominal focused assessment with sonography for triage, trauma, and tracking rapid (AFAST) for hemoabdomen CBC to identify anemia Bronchoscopy for airway assessment

Megaesophagus Megaesophagus is present on this radiograph. It can be idiopathic or secondary to several disorders. It is a frequent finding with myasthenia gravis. Other causes of megaesophagus include esophagitis, esophageal stricture, or diverticulum, heavy metal toxicity, polymyositis, lupus, hypoadrenocorticism. A thorough neurologic examination, complete bloodwork including adrenal testing, and possibly contrast studies of the esophagus should be perform in this patient. Regurgitation is actually present in this dog. This occurs very soon after eating, while vomiting usually occurs a few hours later. Attention to details of the history is important to help direct diagnostics.

A nine-year-old Irish setter is presented for vomiting. The owner states that the dog does not really retch, but just spits up food soon after eating. The dog is normal otherwise. Based on the lateral radiograph below, which one of the following choices is the most likely diagnosis? Persistent right aortic arch Gastric foreign body Megaesophagus Mediastinal mass Pulmonary bullae

Bone spavin This is a bone spavin, an osteoarthritis/degenerative joint disease of the distal hock joints, usually affecting the distal intertarsal and tarsometatarsal articulations. The number one hock problem in horses. If the horse is lame due to bone spavin, it tends to drag toe or have "stabbing" gait in the affected hindlimb. Occasionally you may see thickening of the soft tissues over distal hock joints. The problem can be unilateral or bilateral with various degrees of lameness. Typical TX: intra-articular steroids and hylaruonic acid, phenylbutazone as needed, continue with work. Over time, distal hock joints are likely to ankylose on their own and lameness ends. Additional treatments include shockwave, bisphosphonates, alcohol arthrodesis, cunean tenectomy or neurectomy. Tarsal hydrarthrosis (bog spavin) is a chronic synovitis of the tarsocurual joint - look for swelling and distention of the joint capsule.

A nine-year-old Quarterhorse mare is presented with a right hind limb lameness. She tends to drag the right toe, and has a stabbing gait in that limb (adducts hindlimb, then suddenly abducts it just before the hoof hits the ground). Hock flexion of the right hind significantly exacerbates the lameness for 5-6 strides. A radiograph of the hock is taken, shown below. What is the common name for this problem? A - Tarsal hydrarthrosis B - Osselets C - Ringbone D - Splints E - Bone spavin

Munge This is a clinical picture of munge. Some affected animals have lesions on the bridge of the nose and around the eyes and ears. Average age of onset is six months to two years old. RX - address secondary bacterial infections (iodine). If minimal response, include a topical glucocorticoid or intralesional triamcinolone acetonide. If still no response, evaluate immune function. Dorsal nasal alopecia is characterized by alopecia over the bridge of the nose with normal to variably scaly, hyperpigmented, and thickened skin. It is more common in dark-haired animals. It can be secondary to fly bites or rubbing the nose. Animals with idiopathic hyperkeratosis present with thickened, nonpruritic papules with tightly adherent crusts in the less densely haired areas of the perineum, ventral abdomen, inguinal region, medial thighs, and axilla. Signs can begin at any age.

A one-year-old female llama is presented with hyperkeratosis around the nose and mouth. What is the most likely diagnosis? A - Zinc-responsive dermatosis B - Wry face C - Dermatophilosis D - Dorsal nasal alopecia E - Munge

Panosteitis This is panosteitis, an acute-onset shifting leg lameness with long bone pain most commonly seen in young (5-18mo) medium to large breed dogs. GSD are the highest risk breed. Hypertrophic osteodystrophy (HOD) is an uncommon development orthopedic disease of young (2-8mo), growing dogs. Large and giant breeds, like the Dane, are predisposed. It is characterized by symmetrical distal metaphyseal pain and swelling of long bones (most commonly ulna and tibia), fever, anorexia, and depression. Hypertrophic osteopathy is a diffuse periosteal proliferative disease of long bones in dogs secondary to a neoplastic mass or other disease in the thoracic or abdominal cavity. Multiple cartilaginous exostoses is a rare benign proliferation of bone/cartilage that can affect the metaphyseal cortical surfaces of long bones, vertebrae, and ribs in young dogs and cats. Animals may have no signs at all. Osteosarcoma is unlikely in a young dog and does not cause shifting leg lameness.

A one-year-old female spayed German Shepherd is presented for a two-week history of lameness. Although the dog limps on the right foreleg as it enters the exam room, the owner reports that the dog was lame on the left hindleg last week. On physical exam, the dog reacts painfully on palpation of the long bones of the right foreleg. What is the most likely diagnosis? Values 99.8, 100bpm, 24rr (normal is 99.5-102.5, 80-120, 15-34) A - Panosteitis B - Hypertrophic osteodystrophy C - Hypertrophic osteopathy D - Osteosarcoma E - Multiple cartilaginous exostoses

Lavender foal syndrome This presentation is consistent with LFS, an inherited lethal neurologic disorder that affects Arabian foals with a dilute coat color. LFS is a recessive genetic disorder and newborn foals present as in this case. Affected foals are typically euthanized within days of birth. There is no treatment. A genetic test is available and affected animals should not be bred. Botulism in foals less than 1mo old (shaker foal syndrme) is characterized by progressive symmetric motor paralysis with muscular tremors. Affected foals cannot stand for more than a few minutes at a time. Overo lethal white syndrome is characterized by colic and meconium impaction du agangliosis. It occurs in Paint foals with blue eyes that are the result breeding two coat pattern in Paint horses). Myasthenia gravis is not reported in horses. Neurologic signs seen with hyperammonemia associated with portosystemic vasc (PSVAs, also referred to as portosystemic shunts) are not seen until affected foals old and eating more grain and forage.

A pale-colored Egyptian Arabian foal is presented for evaluation within a few hours of a dystocia with assisted vaginal delivery. The foal has opisthotonos, nystagmus, limb rigidity and paddling, and has not yet stood and nursed. What is the presumptive diagnosis based on the information available? A - Overo lethal white condition B - Lavender foal syndrome C - Botulism D - Myasthenia gravis E - Portosystemic vascular anomaly

Require original owner consent or court order to release information The vet clinic requires the original owner's consent or a court order to release information. "Ethically, the information within veterinary medical records is considered privileged and confidential. It must not be released except by court order or sent of the owner of the patients. Veterinarians should secure a written release to document that request.

A person calls to say she adopted a friend's dog and requests a copy of the dog's medical record and radiographs. The former owner is a client at your clinic. Which one of the following choices is the most appropriate action to take? A - Give a summarized record with personal identifiers (vet, owner names) blocked out B - Give caller a copy of medical record and radiographs C - Require original owner consent or court order to release information D - After physical exam, start a new medical record documenting previous issues E - Give caller a copy of medical record only

This is an inguinal hernia in a piglet. A common problem in pigs; treat surgically via midline skin incision, cranial to scrotum. Important to ligate and excise the vas deferens and blood vessels. Both inguinal rings should be closed to prevent herniation post-surgery. Prevent seroma formation by removing the tunic, cremaster muscle, and extra SQ tissue; surgically close any dead space. Intersex syndrome is described in pigs and goat (rare), but look for male and female genitalia. Perineal hernia is more a problem of middle aged pure-bred dogs. In cryptorchid animals, a testicle is retained, no protruding.

A seven-week-old pot-bellied pig is presented with a posterior swelling that looks like the image below. Which one of the following is the most likely clinical diagnosis? A - Intersex syndrome B - Testicular torsion C - Perineal hernia D - Cryptorchidism E - Inguinal hernia

Ultrasound of the metacarpus Ultrasound of the metacarpus is recommended to further evaluate the soft tissues of the metacarpus. This appearance of a "bowed tendon" is typical of an acute superficial digital flexor tendinitis. Ultrasound is necessary to confirm and to evaluate the extent and severity of the lesion. Nerve blocks are unnecessary and possibly even inappropriate as desensitizing the area and jogging the horse could leads to greater damage of the affected structures. Rx includes stall rest, anti-inflammatories, and ice, and bandaging. Additional therapies include intralesional regenerative therapies or steroids, shockwaves therapy, tendon splitting, and a rehabilitation program. Radiographs and nuclear scintigraphy are used mostly for bony lesions.

A seven-year-old Thoroughbred event gelding is presented after coming up lame on the left forelimb following completion of a course. The horse is 4/5 lame on the left forelimb. There is moderate swelling over the palmar aspect of the left metacarpus as seen in the image below. The horse is sensitive to palpation of the swollen area. Which diagnostic would be most appropriate to recommend? A - No further diagnostics needed - diagnosis is clear B - Ultrasound of the metacarpus C - Diagnostic perineurial anesthesia (a.k.a. nerve blocks) D - Nuclear scintigraphy (a.k.a. bone scan) E - Digital radiographs of the distal limb

Uterus The chinchilla has pyometra. The abdomen is markedly distended with poor detail caused by organomegaly. There are two soft tissue tubular masses in the caudal abdomen along the lateral margins. The small bowel loops are displaced cranially and medially. Within the thorax there are no abnormalities seen.

A seven-year-old female chinchilla is presented with a history of anorexia and lethargy. Which one of the following organs has the most prominent pathology in these radiographs? A - Urinary bladder B - Uterus C - Normal radiographs for a chinchilla D - Stomach E - Colon

Myxomatous mitral valve disease (MMVD) MMVD is the most common heart disease in dogs. It is the most common cause of a heart disease in dogs. It is the most common cause of a heart murmur in adult small- to medium-sized dogs, especially Cavalier King Charles Spaniels, dachshunds, yorkies, chihuahuas, mini poodle. Myxomatous degeneration of the tricuspid valve can also occur. Stage dogs with MMVD to determine the need for intervention - use thoracic radiographs, +/- ECG. Asymptotic dogs with a heart murmur and normal thoracic radiographs are considered class B1. Do not treat class B1 dogs, but monitor for progression. Progression cans be slow to negligible. There is no tx that slows or reverses MMVD, however dogs with cardiomegaly benefit from tx with pimobendan to slow progression of CHF. If CHF is present, tx to alleviate clinical signs and improve quality of life.

A six-year-old male neutered Yorkshire terrier is presented for routine examination. The owner has no current concerns. Exam reveals a grade 3/6 left-sided systolic heart murmur. Pulse quality is strong and synchronous and there are no other abnormalities. What is the most likely cause of the heart murmur? A - Myxomatous mitral valve disease B - Aortic stenosis C - Patent ductus arteriosus D - Carnitine-deficient dilated cardiomyopathy E - Physiologic

Abdominal radiographs There is an enterolith in the manure so abdominal radiographs should be recommended. There if often more than one, and abdominal radiographs are the least invasive way to evaluate for more. Exploratory laparotomy is the definitive method, but is invasive, costly, and not warranted without radiographs and or clinical signs. Enteroliths are composed of magnesium, ammonium, and phosphate and typically form around a nidus of sand or foreign material. Most common in Arabians that eat alfalfa hay in Ca, southwest, TX, and FL, While small stones can be passed, enteroliths more often become too large to pass per rectum. They cause recurrent colic or acute colic when they become lodged in narrow bowel. Definitive treatment - ventral midline celiotomy to remove the stones via enterotomy. Prognosis is excellent.

A ten-year-old Arabian gelding in California is presented because of an interesting finding in his manure. The owner mentions that he's had a couple of mild colic episodes over the past months that resolved with just walking and oral flunixin meglumine administered by the owner. The manure is shown below: What diagnostic should be recommended to the owner to determine how to proceed? Ultrasound per rectum Gastroscopy and duodenal biopsy Fecal occult blood test McMasters fecal flotation on fresh manure Abdominal radiographs

Surgical removal This is a basal cell tumor. Basal cells make up the base of the epidermis, and a benign growth of these is called a basal cell tumor or adenoma. Although this tumor is benign, surgical removal is the treatment of choice due to ulceration and secondary inflammation. Basal cell carcinomas (malignant form) do not form raised masses but appear as ulcers on the head, neck, and legs: surgical removal is also recommended. Basal cell tumors are common in older cats, accounting for 15-26% of all feline skin tumors. Most are slow growing and typically present for months prior to diagnosis. Domestic longhairs, Himalayans, and Persians are predisposed. Note the characteristic cytology findings of epithelial neoplasia present in this case: cohesive clusters of large, round to polygonal cells, demonstrating distinctive cell borders, that are tightly adherent to each other with round to oval nuclei.

A ten-year-old female spayed Persian cat is presented for evaluation of a two-month history of a mass. There is a two-cm diameter, firm, solitary, hairless, dome-shaped, ulcerated mass on her left neck just behind the jaw that is bleeding. Cytology obtained by fine needle aspirate reveals cohesive clusters of large cells with round to oval nuclei that are tightly adherent to each other (see below). What is the most appropriate treatment option? A - Wide excision of mass, remove regional lymph node, prednisolone, radiotherapy B - Immunotherapy, wide margin excision, chemotherapy C - Surgical removal D - Debulk, intralesional implant chemotherapy, minimize sun and cigarette smoke exposure E - Cryosurgery, debridement, repeat in 3 - 5 weeks

Diaphragmatic hernia This cat has a diaphragmatic hernia. There is soft tissue opacification on the right hemithorax. There are irregular gas opacities within the soft tissues of the right hemithorax. The trachea and carina appears displaced to the left. The cardiac silhouette is difficult to evaluate. In the viewable cranial abdomen, the stomach, and liver are not identified.

A ten-year-old male neutered domestic shorthair cat is presented with a history of being hit by a car one month ago. There is a recent onset of weight loss. Lung sounds are muffled on the right side. What is the most likely diagnosis based on interpretation of these radiographs? A - Pleural effusion B - Megaesophagus C - Vascular ring anomaly D - Aspiration pneumonia E - Diaphragmatic hernia

Dermatophilosis Dermatophilosis is a crusting dermatitis caused by the filamentous bacteria Dermatophilus congolensis, seen in all ruminants and horses. Also called lumpy wool or rain scald/rot. Infection is seen most often in younger or systemically ill/immunocompromised animals. Many are housed in areas with poor hygiene or without shelter. Excessive moisture, rainy conditions, skin damage, and concurrent illness that results in immunosuppression are predisposing factors. Loss of the protective sebaceous film layer seen on healthy skin is thought to be an important factor in the pathogenesis. Carrier animals and fomites such as grooming tools are also important as this can contagious in at-risk animals. Thick crusts, which usually contain the organism, look like the end of a paint brush when removed from the animal and are a typical finding. On cytology (impression smear of the crust), the railroad track appearance of the gram-positive cocci is classic for dermatophilosis. The infection self-resolves after a few weeks in most animals without treatment. Can use topical and systemic antimicrobials in persistent cases.

A three-year-old female goat presents in poor body condition, slightly depressed, and febrile. The goat is mildly pruritic. The skin shows mild to moderate erythema and alopecia and marked hyperkeratosis and crusting on all four extremities, as shown in the image below. Skin scrapings are negative for mites. A cytology taken from the underside of a crust shows parallel stacks or rows of cocci ("railroad tracks") between keratinocytes, also shown below. Which one of the following choices is the most likely diagnosis? A - Dermatophytosis B - Streptococcus infection C - Dermatophilosis D - Pyoderma E - Malassezia dermatitis

Treat with potassium permanganate Treat fish with potassium permanganate, hydrogen peroxide, or formalin once causative factors have been addressed for this Saprolegnia infection. Saprolegnia is a saprophytic fungus-like pathogen (oomycete water mold) that commonly causes disease in freshwater fish. A direct smear should reveal large, nonseptate filaments. Low water temperatures are a major predisposing factor for infections. Other possible causes include poor sanitation and the presence of decaying organic material. Products that contain malachite green are effective but may not be used on food fish. A similar orgnaism, Aphanomyces invadans, causes epizootic ulcerative syndrome (EUS) in fish. EUS is a reportable disease, endemic to much of US and immediately notifiable in Canada. Saprolegnia and the Aphanomyces oomycetes can cause huge economic losses in farmed fish. Crayfish have their own oomycetes, A. astaci. Through endemic in North American crayfish, it does not generally cause clinical disease. European and Australian crayfish are vulnerable to death and disease.

A trout fishery reports that many of the fish have gray-white, puffy growths on their fins, gills, and eyes. A direct smear from affected tissues shows the following findings (images below). What action should you take to address the top differential? A - Administer praziquantel to all the fish B - Supplement the fish with vitamin C C - Ask if there is any zinc in the housing D - Depopulation is the only appropriate measure E - Treat with potassium permanganate

Due to colostrum absorption Very high GGT levels are seen with colostrum absorption. In newborn pups, lambs, and calves, and increases in GGT up to 1000x is normal when they receive high levels of colostrum from the dam. High levels of GGT are produced in the mammary epithelium during lactation. When evaluating bloodwork, reference ranges specific for the age of the patient should be utilized.

A two-day-old pup has a GGT level of 500 times the adult upper reference range. Which one of the following choices is the most likely interpretation? A - Related to bone development B - Associated with neonatal cholestasis C - Caused by traumatic placental detachment D - Due to colostrum absorption E - Suggestive of a congenital liver shunt

Conium maculatum (poison hemlock) Poisoning with Conium maculatum is responsible for the death of this cow. At least eight toxic piperidine alkaloids have been isolated from poison hemlock. Coniine is found in seeds and the mature plants; g-coniceine is found in young growing plants Poison hemlock is toxic to all livestock and humans. Signs of toxiity develop 1-2 hours after ingestion and are usually fatal. Signs include nervousness, trembling, weakness (especially hind limbs), weak pulsa, irregular heart rate, recumbency, coma and death. A mousy odor exuding from the urine and breath is pathognomonic. Ingestion of poison hemlock during gestation causes athrogryposis and other congenital defects in cattle, goats, and pigs.

A two-year-old Angus cow is presented with weakness of the hind limbs and a staggering gait. On physical exam, the heart rate and respiratory rate are slow and irregular, and the pupils are dilated. The cow's breath and urine smell like the odor of mouse urine. Suddenly the cow's pulse becomes rapid and thready, she collapses, and dies of apparent respiratory failure. Which one of the following plants is most likely to have caused this spectrum of clinical signs? Pinus ponderosa (Western yellow pine) Veratrum spp (False hellebore, Skunk cabbage) Tetradymia spp (Horsebrush) Conium maculatum (Poison hemlock) Centaurea spp (Knapweed, Yellow star thistle)

Sarcoptes scabei This is Sarcoptes scabei, a highly contagious mite. A positive pinnal-pedal reflex is seen in 82% of dogs with sarcoptic mange, but observed in only 7% of dogs with pruritus caused by other diseases. Cheyletiellosis (walking dandruff) is another highly contagious mite. Unlike sarcoptes, they have very prominent mouthparts.

A two-year-old beagle is presented with patchy alopecia, papules, and thick crusts, on the ventral abdomen, chest, ears, and elbows. The owner reports intense pruritus and a positive pinnal-pedal reflex is observed. A flea comb does not reveal any fleas. The image below shows what was discovered from a skin scraping. Which one of the following organisms is depicted in the image? A - Neotrombicula autumnalis B - Straelensia cynotis C - Demodex canis D - Sarcoptes scabei E - Cheyletiella yasguri

Paired plasma and effusion potassium and creatinine. This cat has a traumatic urinary bladder rupture (uro-abdomen), best diagnosed with paired serum and effusion potassium and creatinine measurements. Uro-peritoneum is a surgical emergency. In cats, a fluid to blood creatinine ratio of 2:1 and fluid to blood potassium ratio of 1.9:1 is diagnostic for uroabdomen. BUN measurements is often not helpful because it readily equilibrates across the peritoneum. High rise syndrome is a common polytrauma in cats, characterized by a triad of thoracic, craniofacial, and limb injuries. Stabilize life-threatening thoracic trauma such as pneumothorax, pulmonary contusions, and diaphragmatic hernia before surgical treatment of orthopedic trauma. Uroperitoneum also occurs in newborn foals. Affected foals typically present within a couple days of birth with abdominal straining and distention, tachypnea, and decreased nursing with depression. Labs - hyponatremia, hypocholremia, hyperkalemia, azotemia

A two-year-old male neutered domestic shorthair cat is presented after a suspected fall from a height of approximately 35 feet (over ten meters). The cat is initially stabilized with intravenous fluids and oxygen therapy. Four hours later, the cat has not urinated and no urinary bladder is palpable. An abdominal focused assessment with sonography for trauma, triage, and tracking (AFAST) reveals a moderate volume abdominal effusion, with no visible urinary bladder. What test would confirm the top differential? Abdominocentesis with Gram stain and cell count Total protein and microscopic analysis of the ascites Lactate and glucose levels of peritoneal sample and blood Measurement of BUN in blood and fluid Paired plasma and effusion potassium and creatinine

Toxocara cati These are adult T. cati, large roundworms that are common in kittens and puppies. Clinical presentation is as in this cat, and worms can be seen in vomit or feces. Affected animals may have mucoid diarrhea or - with early infection - coughing due to larval migration causing eosinophilic pneumonia. Kittens are infected by eating embryonated infective eggs or - rarely - by transmammary transmission. DX: identify adult worms in vomit or feces or eggs in feces. Must distinguish the spherical pitted-shelled eggs of T canis from T cati from the oval, smooth shelled eggs of T leonina. RX: pyrantel, milbemycin, moxidectin, piperazine, emodepside, fenbendazole, or selamectin RX queen in last of pregnancy, then TX nursing queen and kittens. Eggs are sticky and making it difficult to decontaminate the environment. ZOONOTIC - humans can develop visceral or ocular larval migrans Strongyloides stercoralis adults are only ~2mm in length and almost transparent Trhichuris vulpis are 45-75mm long and slender with a thicker posterior. Ancylostoma tubaeform are 12-15 mm long. Spirocerca lupi are bright red and 40-70mm long.

A young cat with a pot belly and dull coat has vomited up a hairball that contains a large tapeworm and many smaller worms. What are the smaller parasites? A - Trichuris vulpis B - Ancylostoma tubaeforme C - Strongyloides stercoralis D - Spirocerca lupi E - Toxocara cati

Limb amputation Limb amputation is the treatment of choice in this dog with osteosarcoma. Also consider adjunctive chemotherapy due to the likelihood of metastatic disease at the time of diagnosis. On radiographic review, the lesions are characteristic of a primary bone tumor. There is a large, expansile, destructive osseous lesion of the distal right tibia with cortical destruction, most severe at the caudal and medial margins. There is also mild, irregular periosteal proliferation and soft tissue thickening at the site. Osteosarcoma is a top differential for an older large-breed dog with an expansile, destructive osseous lesion.

An eight-year-old female Saint Bernard is presented with a six-week history of right pelvic limb lameness and swelling above the hock. A radiograph of the affected limb is shown below. Three-view thoracic radiographs are unremarkable. Based on the top differential, what is the treatment of choice? A - Lipid complex amphotericin B IV B - 6 - 8 weeks of oral clindamycin C - Limb amputation D - Intravenous cyclophosphamide E - Intralesional mitomycin D

Bovine tuberculosis These are the classic granulomas and abscesses of bovine tuberculosis. Most commonly caused by Mycobacterium bovis or Mycobacterium tuberculosis in cattle. TB is reportable. Look for cows with chronic debilitation, emaciation, but without the telltale watery diarrhea of Johne's disease. It is transmitted via aerosol route. Actinobacillosis causes wooden tongue in cows.

An emaciated cow is presented. She appears weak and anorexic, with enlarged superficial cervical lymph nodes. TPR: 102.9, 60 bpm, 30 bpm The cow dies the next day. On necropsy, the following lesions in the lungs are evident. What is the diagnosis? A - Bovine tuberculosis B - Bovine lymphoma C - Actinobacillosis D - Johne's disease E - Contagious bovine pleuropneumonia

Bovine tuberculosis These are the classic granulomas and abscesses of bovine tuberculosis. Most commonly caused by Mycobacterium bovis or Mycobacterium tuberculosis in cattle. TB is reportable. Look for cows with chronic debilitation emaciation, but without the telltale watery diarrhea of Johne's disease. It is transmitted via aerosol route. Actinobacillosis caused wooden tongue in cows

An emaciated cow is presented. She appears weak and anorexic, with enlarged superficial cervical lymph nodes. The cow dies the next day. On necropsy, the following lesions in the lungs are evident. What is the diagnosis? A - Bovine tuberculosis B - Johne's disease C - Contagious bovine pleuropneumonia D - Actinobacillosis E - Bovine lymphoma

Edema disease Edema disease is caused by shiga toxin producing E. coli (STEC). Look for severe acute illness ranging from peracute death with no signs to CNS involvement with ataxia, paralysis, and recumbency in healthiest pigs 1-2 weeks after weaning. Streptococcus suis could cause sudden death but would cause diarrhea and would see polyserositis on necropsy. Clostridium perfringens type C enteritis, also called enterotoxemia in other animals, is characterized by a hemorrhagic diarrhea in 1-3 day old piglets. Porcine proliferative enteritis is principally a diarrheal disease of growing finishing (40-80lb) pugs and young breeding pigs. Epidemic transmissible gastroenteritis (TGE) in non-immune pig herds characterized by HIGH MORBIDITY and HIGH MORTALITY in piglets less than 1 week old.

An outbreak of diarrheal disease of piglets has occurred which affected the healthiest animals in the herd, one to two weeks after weaning. Some affected piglets had no signs except peracute death. Other affected piglets exhibit diarrhea, ataxia, paralysis, and recumbency. What condition is at the top of the differential diagnosis list? Epidemic transmissible gastroenteritis (TGE) Streptococcus suis Clostridium perfringens type C enteritis Edema disease Porcine proliferative enteritis

Fever, petechiations, hemolytic anemia. This cytology shows Babesia caballi, one of the causative agents of equine piroplasmosis (other is Theileria equi - looks more like a maltese than the apple seeds). Piroplasmosis is characterized by hemolytic anemia and is spread primarily by ixodid ticks or blood contamination. It is endemic in tropical, subtropical, and some temperate regions. IT IS REPORTABLE IN THE USA Clinical infection can be acute, chronic, or inapparent, depending on the strain, the horse's health, and geographic region. Acute clinical signs include those described here along with lethargy, anorexia, pale or icteric mm, and with worsening disease - tachycardia, tachypnea, weakness, and hemoglobinuria. Weight loss, poor performance, and partial anorexia can be seen along with mild anemia in horses with chronic piroplasmosis. DX - demonstrate intra-erythrocytic organisms on blood smear and/or serology RX - Imidocarb dipropionate and supportive care. There is a different duration of TX depending on whether in endemic or nonendemic region (treat clinical signs versus clear disease). Limb edema, ventricular arrhythmias, and ataxia (the latter two uncommon) can be seen with Anaplasma phagocytophilia infection in horses, along with fever, petechiations, icterus, and depression.

Cytological evaluation of a blood smear from a sick horse reveals the following (see below). Which constellation of signs would you expect in a typical acute case of a horse with this disease? A - Nystagmus, head tilt, strabismus B - Weight loss, partial anorexia, poor performance C - Elevated serum liver enzymes, hyperammonemia, head-pressing D - Limb edema, ventricular arrhythmias, ataxia E - Fever, petechiations, hemolytic anemia

Liver and muscle In dogs and cats, damage to liver and muscle cells may cause increases in serum alanine aminotransferase (ALT). ALT is a "leakage' enzyme, meaning that the enzyme is usually intracellular. It leaks out of cells and ultimately into circulation following damage to the cell membrane. In other species (horses, ruminants, pigs, birds), ALT is less useful because there are not high intracellular levels. Aspartate aminotransferase (AST), sorbitol dehydrogenase (SDH), and glutamate dehydrogenase (GLDH) are the predominant leakage enzyme in these animals (availability is lab-dependent). The magnitude of increase of leakage enzymes may be deceiving. Severe damage to a healthy liver may result in very high levels, while low levels may be seen when significant atrophy or fibrosis of the liver is present and few cells are left. Differentiate ALT from muscle vs liver by evaluation of the clinical picture and other serum biochemistry results. Other muscle enzymes include creatine phosphokinase (CK), and AST; and other liver enzymes are AST, SDH, GLDH, gamma glutamyl transferase (GGT), and alkaline phosphatase (ALP). Leakage enzymes do NOT provide an estimate of liver function. This is evaluated by measurement of substance dependent on the efficacy of liver function such as bile acids, blood urea nitrogen, albumin, and glucose.

Damage to which tissues can cause increases in serum alanine aminotransferase (ALT) levels in dogs and cats? A - Pancreas and intestine B - Red blood cells and brain C - Liver and muscle D - Spleen and adrenals E - Heart and kidneys

Normal finding Normal male crias are born with adhesions between the free end of the penis and the prepuce. As they sexually mature, these adhesions gradually detach so puberty can be seen clinically when the penis can be completely exteriorized. Castration prior to puberty may result in continued adhesions. In Peru, males are not generally used for breeding until 3 yrs. Male llamas reach puberty at approximately 21 months (9-31 months). Alpacas mature earlier, around 12 months of age.

During examination of a newborn male cria, adhesions between the free end of the penis and the prepuce are evident. What is the most likely explanation? A - Common congenital malformation of preputial ring, associated with phimosis B - Failure of separation of preputial diverticulum C - Sequella of hypospadias; Heritable D - Normal finding E - Secondary to stress in utero, 5th month

Alterations in diet Foal heat diarrhea is associated with alterations in diet and changing microbial flora in the GI tract. Mild, self-limiting diarrhea in a foal 7-14 days of age is called foal heat diarrhea because it coincides with the first estrus cycle post-foaling in the dam. Causes of foal heat diarrhea are poorly understood, but are thought to be related to a foal's tendency to start sampling hay and grain and practice coprophagy by 5 to 7 days of age, with consequent alterations in microbial flora. Affected foals are active and alert, with a normal appetite and vital signs. Clinical signs such as fever and lethargy, hematochezia, or melena, and laboratory findings such as neutropenia are not routinely observed.

Foal heat diarrhea is typically associated with which of the following? A - Hemorrhagic enteritis B - Neutropenia and fever C - Warm and humid weather D - Decreased suckling E - Alterations in diet

PVP decreases as prevalence decreases As prevalence of disease goes down, PVP of your serologic test also goes down. That is, as your disease becomes more and more rare, the predictive value of your same old test gets worse. That is all you need to know.

For the last ten years, your state has had a mandatory vaccination program against "pedunculated giblet disease" in fur-bearing turtles and the prevalence of this terrible disease has decreased markedly. How does this decrease in prevalence affect the predictive value positive (PVP) of the best serologic test for pedunculated giblet disease? A - PVP increases as prevalence decreases B - PVP decreases as prevalence decreases C - PVP stays the same as prevalence decreases D - PVP is affected by specificity, not prevalence E - PVP depends on the number tested, not prevalence

120-150 days An experienced practitioner can reliably feel fremitus between 120 and 150 days. Remember, fremitus is supportive sign of pregnancy, but not conclusive evidence of pregnancy. An experienced practitioner can reliably feel the membrane slip of chorioallantoic membranes rectally around 30-35 days and can feel placentomes at around 70-75 days. Placentomes and membranes slip are cardinal signs of pregnancy. S - slip 30-35 P - placentomes - 70-75 F - fremitus - 120-150

What is the earliest stage post-breeding that an experienced practitioner can reliably rectally palpate fremitus in the uterine artery of the gravid uterine horn in a pregnant cow? A - 90-120days B - 60-75 days C - 120-150 days D - 70-80days E - 45-60 days

Closure of the atrioventricular valves, opening of semilunar valves. First heart sound (S1/lub) is caused by closure of the AV valves and opening of the semilunar valves. This is the end of diastole and start of systole. Second sound (S2/dub) is the closure of the aortic and pulmonic valves and opening of the AV valves. This is the end of systole and start of diastole. A third sound (S3) is the end of rapid ventricular flling and a fourth sound (S4) is atrial systole/contraction. You often hear all 4 sounds in horses, but typically hear only S1 and S2 in cattle and small animals.

One description of a typical heart sound is "lub-dub." What makes the first heart sound (S1) (i.e., the "lub")? A - Atrial contraction B - Rapid atrial and auricular expansion due to cranial and caudal vena cavae contraction C - Aortic and pulmonic valves shut and the mitral and tricuspid valves open D - Closure of atrioventricular valves, opening of semilunar valves E - End of rapid ventricular filling period

Bluetongue Swollen sore muzzles with mucous membrane erosions, high fevers, and lameness suggests infection with bluetongue virus. Bluetongue is almost exclusively seen in sheep, though white-tailed deer, pronghorn antelope, and desert bighorn sheep in North America can be severely affected. Rare in cows - REPORTABLE Bluetongue is clinically indistinguishable from foot and mouth disease (FMD), so that is a good second choice. But FMD is unlikely in the scenario above because FMD mainly affects pigs and cattle. Ovine progressive pneumonia (OPP) is a chronic disease of wasting and dyspnea in sheep and goats, caused by a lentivirus (retrovirus family). OPP is most common in animals older than four years and rarely occurs in sheep and goats under two years of age.

Several 12-month-old sheep and goats are sick at a petting zoo that has cows, horses, pigs, bison, and white-tailed deer. One of the deer is also affected. The rest of the animals appear well. Affected sheep are febrile, listless and off-feed, with serous or mucopurulent nasal discharge and rectal temperatures ranging from 105-107.5°F (40.5-42°C) [N=100.9-103.8°F (38.3-39.9°C)]. Physical exam shows swollen muzzles with edema and congestion of the lips, nose, and face with small hemorrhages and ulcers on the mucous membranes. The ulcers appear where the teeth contact the swollen lips and tongue. Two affected animals are lame. What is the diagnosis? A - Peste des petits ruminants (PPR) B - Ovine progressive pneumonia (OPP) C - Bluetongue D - Caprine arthritis encephalitis (CAE) E - Contagious ecthyma

Chloroquine This presentation is consitent with "velvet disease", a parasitic dinoflagellate infection by Piscinoodinium spp. in freshwater fish and Amyloodinium spp. in saltwater species. The treatment in ornamental fish is typically chloroquine or copper sulfate, given as a bath or in quarantine tank. The treatment in fish raised for food in the US is repeated treatments of copper sulfate - similar to Ich (Ichythyophthirius multifiliis). Velvet disease is one of the biggest health concerns in captive marine fish, especially clownfish, and can look like white spots. The nonmotile, photosynthetic, parasitic algae attaches to the skin and gills. There, it invades the tissues and gives rise to cysts that contain flagellated, free-swimming stages of the parasite that go on to initiate new infections. The prognosis is guarded because this disease is difficult to manage. Potassium permanganate and hydrogen peroxide are treatments for oomycete infections in fish (primarily fresh-water). Clinical presentation includes gray-white, cotton-like growths on the skin, eyes, gills, or fins. Tricaine methanesulfonate (MS222, Tricaine mesylate) is the only anesthetic licensed in the United States

Several ornamental freshwater fish (Betta splendens) have recently developed a velvety, rust-colored appearance, especially around the eyes and gills (see below). What is the appropriate treatment in pet fish for the presumptive diagnosis? A - Hydrogen peroxide B - None; this condition is self-limiting C - Potassium permanganate D - Chloroquine E - Tricaine methanesulfonate (MS222)

Vaccinate all animals with Clostridium novyi toxoid Active immunization with Clostridium novyi toxoid before the late summer is the most effective way to control and prevent infectious necrotic hepatitis (Black disease) in sheep. Because vaccinated sheep have long-term immunity after only one shot, only new introductions to the flock (lambs and sheep brought in from other areas) need to be vaccinated. Reducing the number of snails (intermediate hosts for fluke cercaria) with molluscicides or by fencing off wet areas may not be practical due to expense or amount of the pasture lost. likewise, use of flukicides is complicated by the need for careful timing and long withdrawal times for meat and milk.

Several younger sheep in a large herd have died suddenly with a diagnosis of infectious necrotic hepatitis secondary to fluke infestation. Which choice is the most practical and effective control measure you can take next to prevent future cases? A - Clostridium hemolyticum bacterin for animals under 3 years B - Vaccinate all animals with Clostridium novyi toxoid C - Treat ponds with copper sulfate molluscicide against lymnaeid snails D - High-dose penicillin for clinical cases E - Clorsulon antihelmintic for whole herd

IV Fluids, NSAIDs, and frequent milk feedings are indicated. Fever, diarrhea and sudden death in either week old calves is highly suggestive of septicemia due to salmonellosis. Isolate sick calves. Use of antibiotics is controversial as they may prolong recovery and shedding and yield a carrier calf, however if an animals is septic, it needs parenteral antibiotics. Prognosis is poor with neonatal salmonella and deaths can approach 100% in affected calves. In adults, antibiotics may yield a clinical cure, but Salmonellae can establish in the biliary system and intermittently shed into the GI system, leading to environmental contamination. Prevention is dependent on which species of salmonella is causing the problems - host adapted or environmental,

Two eight-week old calves are presented down and extremely weak. They are depressed and lying in pools of foul-smelling brown diarrhea with a small amount of blood. Another calf died suddenly the previous night with no prior signs. The down calves are dehydrated, with rectal temperatures of 105.2° and 105.6° F (40.7° - 40.9° C) [N=101.5°-103.5°F, N=37.8°-39.7° C], respectively. What is the treatment plan? IV fluids; NSAIDS; frequent milk feedings Cull sick calves; prophylactic oxytetracycline in feed for well animals Isolate sick calves; oral electrolytes Corticosteroids; amprolium in water; rumensin in feed Immunize calves and adult cattle with MLV vaccine; probiotics for sick calves

Rare disease - early diagnosis improves prognosis You need a very sensitive test if: 1. Disease is rare 2. Early DX improves prognosis 3. Disease is highly lethal or consequences of missing a case are severe Remember that a highly sensitive test will have very few false negatives. That means if a test is highly sensitive, you can trust a negative test. Sensitivity = a/(a+c) (a = true positive, c = false negative) If sensitivity is high, then C must be small Therefore, high sensitivity means you can really trust a negative result to be correct.

Under what conditions is a very sensitive test used? A - Rare disease; early diagnosis improves prognosis B - Common disease; infectious diseases C - Treatment does not affect prognosis; non-infectious diseases D - Lethal disease; highly prevalent disease E - Zoonoses; untreatable diseases

Salmonella typhimurium Salmonella typhimurium is associated with rectal strictures in growing pigs. Caused by an ulcerative proctitis that damages rectal tissue Can see large numbers of cases. Can see sporadic rectal strictures as a sequelae to rectal prolapse. Intestinal spirochetosis is a post-weaning diarrhea seen in the absence of Brachyspira hyodysenteriae (swine dysentery), but similar in presentation to it. This syndrome is being recognized more frequently worldwide.

What condition is associated with rectal stricture in pigs? Salmonella typhimurium Rotavirus Coccidiosis Intussusception Intestinal spirochetosis

Chronic external blood loss Chronic external blood loss is the most common cause of iron-deficiency anemia. Dietary iron or copper deficiency or excessive zinc may also cause iron deficiency anemia. Look for microcytic, hypochromic anemia which may be regenerative or non-regenerative. Also look for evidence of RBC fragmentation (acanthocytes, schistocytes, keratocytes) The heme group of hemoglobin contains iron and therefor deficiency causes decreased hemoglobin production. The most common cause of iron deficiency anemia is chronic external blood loss.

What is the most common cause of iron deficiency? A - Immune-mediated hemolytic anemia B - Renal disease C - Dietary zinc deficiency D - Chronic external blood loss E - Copper toxicosis

Mare will need an episiotomy before foaling Episiotomy is a surgically planned incision on the perineum and the posterior vaginal wall prior to foaling. Sometimes may be needed prior to breeding by stallion if the remaining vulvar cleft is too small to permit intromission. A Caslick operation (vulvoplasty) is used to treat pneumovagina in horses to prevent genital infections and infertility.

What recommendation should be given to the owner of a mare that has had Caslick's vulvoplasty surgery? A - Higher risk of uterine torsion B - Mare has a higher than normal risk for metritis C - Mare has a decreased risk for perineal laceration D - Mare will need an episiotomy before foaling E - Mare should not be bred by a stallion

Ancylostoma spp Hookworms may cause CLM in people. Note that roundworms are also zoonotic, causing visceral and ocular larva migrans in people. In dogs, whipworms, are associated with hypadrenocorticism-like syndrome. Whipworm infection has been suggested as one cause of cecocolic intussusception. Habronema spp in horses can cause tumorlike stomach nodules and sometimes cutaneous lesions. Trichostongylus spp cause parasitic gastritis and enteritis in sheep, goats, and cattle.

Which canine parasite can cause cutaneous larva migrans in people? A - Spirocerca sp. B - Ancylostoma spp. C - Trichostrongylus spp. D - Habronema spp. E - Trichuris spp.

Grain overload, ethylene glycol toxicity, renal failure Think of an increased anion gap with renal failure, ethylene glycol toxicity, lactic acidosis from grain overload or strenuous exercise, and diabetic ketoacidosis. AG is an assessment of acid-base status and evaluates unmeasured anions such as ketones, lactate, uremic metabolites, and toxins. Anion gap = (Na + K) - (HCO3 - Cl) If venous blood gas analysis is unavailable substitute TCO2 for HCO3.

Which diseases are associated with a high anion gap (AG)? A - Grain overload, ethylene glycol toxicity, renal failure B - Hypercalcemia of malignancy, hypoadrenocorticism, displaced abomasum C - Eosinophilic enteritis, hypothyroidism, gastrinoma D - Prostatic adenocarcinoma, pleural effusion, lymphangiectasia E - Diabetes mellitus, pancreatitis, chronic diarrhea

Old, small-breed dogs, Cavalier King Charles Spaniel This is endocardiosis (degenerative valve disease). Note the nodular thickening of mitral valve margins. Degenerative valve disease is the most common cardiac disease of dogs, account for about 75% of all canine cardiovascular disease. Roughly 60% of affected dogs have myxomatous degeneration oft the mitral valve. Older, small-breed dogs have a higher incidence and CKCS are prone. Don't confuse endocardiosis (DVD) with infective endocarditis, which is due to bacterial infection of the cardiac valves and endocardial muscle.

Which group is most at risk of developing the lesion evident in this image? Young horses ingesting Perilla mint; Standardbreds Cattle without reticular magnets; Charolais Obese cats, any breed; Burmese cats Old, small-breed dogs; Cavalier King Charles Spaniel Sheep or goats pastured above 2000 meters; Saanens

Basophil - allergy and parasitic infection This is a basophil, typically associated with allergy or parasite infestation. Basophils (BSO), eosinophils (EOS), and neutrophils (PMN) are the three types of granulocytic white blood cells. Phagocytosis is an important function of EOS and PMNs but this is not a primary role of BSOs. Granulocytic phagocytes have segmented nuclei and cytoplasmic granules - orange/pink in EOS; purple/dark blue in BSOs; pale pink in PMNs. The granules of basophils contain many mediators of inflammation and large amounts of histamine. They commonly respond to allergic conditions and parasite infestation, similar to eosinophils.

Which of the following choices correctly identifies the leukocyte in this image of a blood smear from a horse, and the most common types of diseases to which it responds? A - Monocyte; viral/bacterial infection, and chronic inflammation B - Neutrophil; wound repair and bacterial infection C - Basophil; allergy and parasitic infestation D - Lymphocyte; immune disease and viral infection E - Macrophage; bacterial and viral infection

Inability to close the mouth Idiopathic trigeminal neuritis, due to inflammation of cranial nerve 5, is characterized by acute onset of flaccid jaw paralysis. Affected animals cannot close their mouth and have difficulty eating and drinking. Seen occasionally in dogs, rare in cats. Cause is unknown. Idiopathic facial nerve paralysis, affecting CN 7, results in the inability to move the eyelid, lip, or ear and dryness of the eyes and mouth. Masticatory myositis is characterized by pain on opening the mouth and swelling of the muscles of mastication or atrophy of the temporalis and massester muscles with the inability to open the mouth due to fibrosis (acute vs chronic). Dysphagia, dysphonia, and stridor are most often associated with dysfunction of the vagus nerve - CN 10.

Which one of the following choices includes the cardinal sign of trigeminal neuritis? Inability to close the mouth Dysphagia, dysphonia and stridor Circling and head tilt toward side of lesion, no other signs Masseter muscle pain associated with chewing Paralyzed eyelid, ear or lip on one or both sides of the face

Cannot tell if either kid is truly CAE positive or negative You cannot tell if either kid is truly CAE positive or negative yet. Should probably retest both after 90 days, or at next semiannual herd testing. A positive ELISA test for caprine arthritis encephalitis (CAE) in a kid less than 90 days old may reflect maternal antibody or a false positive. A negative test in a kid less than 90 days old does not rule out CAE (though most kids infected at birth from colostrum develop a measurable antibody response by 4-10 weeks after infection) Note lots of qualifiers about CAE testing: A positive test in an adult goat implies infection, but does not confirm that clinical signs were caused by CAE virus. Negative tests do NOT rule out CAE infection. Definitive DX is to find characteristic lesions by biopsy/necropsy. Virus isolation can substantiate Dx, but may not be practical on herd basis.

You are helping a local goat dairy owner interpret the results of semi-annual ELISA serological testing of her herd for caprine arthritis encephalitis (CAE). One two-month-old kid is CAE-positive by ELISA. Another two-month-old kid is CAE-negative by ELISA. How do you interpret these results? A - CAE-positive kid has CAE, cannot say if CAE-negative kid is infected or not B - Both kids are infected with CAE C - Cannot tell if either kid is truly CAE positive or negative D - CAE-positive kid has CAE, CAE-negative kid does not have CAE E - Cannot say if CAE-positive kid has CAE, CAE-negative kid is not infected


Conjuntos de estudio relacionados

Ch 27_ Child with Cerebral Dysfunction

View Set

Pentest+ Lesson 11 - Targeting Mobile Devices

View Set

BIO-1106 Final Exam Practice Questions

View Set

English Questions and Spanish answers

View Set

Chapter 14: Preoperative Nursing Management

View Set

বাংলাদেশের মুক্তিযুদ্ধ ও স্বাধীনতা

View Set

Zoology Unit 2 - Phylum Cnidaria

View Set

Praxis Core academic Skills for Educators: Mathematics Practice Test 2

View Set